TZALOA Revista de la Olimpiada Mexicana de Matematicas ... · de un tiempo, Sissa, le presento´ a...

83
TZALOA Revista de la Olimpiada Mexicana de Matem´ aticas no 2019, No. 1 Comit´ e Editorial: Eugenio Daniel Flores Alatorre Luis Eduardo Garc´ ıa Hern´ andez Jos´ e Antonio G´ omez Ortega Isabel Cristina Mart´ ınez Alvarado Carlos Jacob Rubio Barrios

Transcript of TZALOA Revista de la Olimpiada Mexicana de Matematicas ... · de un tiempo, Sissa, le presento´ a...

Page 1: TZALOA Revista de la Olimpiada Mexicana de Matematicas ... · de un tiempo, Sissa, le presento´ a su rey el juego que le hab´ıa pedido: el ajedrez. Lue-go de entender el juego

TZALOA

Revista de la Olimpiada

Mexicana de Matematicas

Ano 2019, No. 1

Comite Editorial:

Eugenio Daniel Flores Alatorre

Luis Eduardo Garcıa Hernandez

Jose Antonio Gomez Ortega

Isabel Cristina Martınez Alvarado

Carlos Jacob Rubio Barrios

Page 2: TZALOA Revista de la Olimpiada Mexicana de Matematicas ... · de un tiempo, Sissa, le presento´ a su rey el juego que le hab´ıa pedido: el ajedrez. Lue-go de entender el juego

Comite de la Olimpiada Mexicana de Matematicas

Cubıculo 201

Departamento de Matematicas

Facultad de Ciencias, UNAM

Circuito Interior s/n

Ciudad Universitaria

Coyoacan C.P. 04510

Ciudad de Mexico

Telefono: (55) 56-22-48-64

www.ommenlinea.org

Editor: Carlos Jacob Rubio Barrios

Facultad de Matematicas

Universidad Autonoma de Yucatan

Coordinador editorial: Jose Antonio Gomez Ortega

Departamento de Matematicas

Facultad de Ciencias, UNAM

©Queda estrictamente prohibida la reproduccion parcial o total por cualquier sistema

o metodo, mecanico o electronico, sin autorizacion previa del autor.

Impreso y hecho en Mexico.

Febrero de 2019.

Page 3: TZALOA Revista de la Olimpiada Mexicana de Matematicas ... · de un tiempo, Sissa, le presento´ a su rey el juego que le hab´ıa pedido: el ajedrez. Lue-go de entender el juego

Contenido

Presentacion IV

Artıculos de matematicas: Ajedrez y Matematicas 1

Problemas de practica: Examen de invitacion a la OMM, 2018 15

Soluciones a los problemas de practica 22

Problemas de Entrenamiento 27

Problemas de Entrenamiento. Ano 2019 No. 1 27

Soluciones a los Problemas de Entrenamiento. Ano 2018 No. 2 28

2a Olimpiada Mexicana de Matematicas para Educacion Basica 34

Pruebas Individuales 36

Pruebas por Equipos 42

Soluciones de las Pruebas Individuales 45

Soluciones de las Pruebas por Equipos 54

32a Olimpiada Mexicana de Matematicas, Concurso Nacional 61

Apendice 71

Bibliografıa 74

Page 4: TZALOA Revista de la Olimpiada Mexicana de Matematicas ... · de un tiempo, Sissa, le presento´ a su rey el juego que le hab´ıa pedido: el ajedrez. Lue-go de entender el juego

Presentacion

Tzaloa1, la revista oficial de la Olimpiada Mexicana de Matematicas (OMM), es una

publicacion trimestral editada por la Sociedad Matematica Mexicana (SMM). Los artı-

culos, problemas, soluciones, examenes y demas informacion que en ella encontraras,

fueron seleccionados con el fin de apoyar a profesores y estudiantes de nivel medio

superior que cada ano se preparan para participar en los distintos concursos de ma-

tematicas que se realizan dentro y fuera de nuestro paıs.

Ademas de ello, Tzaloa es una publicacion de interes para un publico mas amplio.

Aunque esta concebida para satisfacer las necesidades de la comunidad olımpica, su

columna vertebral es la resolucion de problemas, por lo que tambien resulta de gran

valor para todo aquel que guste de hacer matematicas. El enfoque centrado en los ra-

zonamientos, el contenido expuesto con rigor pero sin formalismos innecesarios o ex-

cesivos, ası como su tendencia al uso de matematica simple y elegante, son algunas de

las caracterısticas que hacen del material expuesto un recurso valioso para profesores,

estudiantes, aficionados y hasta profesionales de las matematicas.

Tzaloa, Ano 2019, Numero 1

Tzaloa recibe el ano con optimismo e inicia su undecimo ano de publicaciones trimes-

trales ininterrumpidas. La consistencia de su publicacion en el contexto nacional, es

un ejemplo de la gran generosidad de muchos profesores y estudiantes que con su tra-

bajo comprometido contribuyen al proyecto. Ası, queremos dar la bienvenida a Isabel

Cristina Martınez Alvarado quien ahora se integra al Comite Editorial de la revista.

Asimismo, aprovechamos la ocasion para agradecer y dar una afectuosa despedida a

nuestro amigo Julio Cesar Dıaz Calderon, quien participo en este comite en los anos

2016, 2017 y 2018 y, a quien le deseamos mucho exito en sus nuevos proyectos.

Pasando al contenido, destaca el artıculo Ajedrez y Matematicas, de Victor Manuel

Grijalva Altamirano. En el, se aborda una seleccion de problemas que involucran un

tablero de ajedrez y se muestran algunas tecnicas de combinatoria para resolverlos.

1Vocablo nahuatl cuyo significado en Espanol es aprender.

Page 5: TZALOA Revista de la Olimpiada Mexicana de Matematicas ... · de un tiempo, Sissa, le presento´ a su rey el juego que le hab´ıa pedido: el ajedrez. Lue-go de entender el juego

Presentacion V

Como en cada numero, hemos puesto todo nuestro entusiasmo en la integracion de las

diferentes secciones que conforman la revista. Todos los problemas, soluciones, exame-

nes y demas contenidos han sido escogidos, revisados y preparados especialmente pen-

sando en el lector. De tal forma, que estando todo listo, solo nos queda desear que todos

nuestros lectores tengan un feliz y prospero ano 2019.

Mexico y las Olimpiadas de Matematicas

Desde sus inicios la Sociedad Matematica Mexicana ha venido impulsando vigorosa-

mente los trabajos de la Olimpiada Mexicana de Matematicas (OMM). Este programa

solo es posible gracias a la participacion de miles de jovenes estudiantes y a la entu-

siasta colaboracion de muchos profesores quienes, de manera espontanea y altruista,

han dedicado sus esfuerzos a mejorar la ensenanza y elevar la cultura matematica de

nuestro paıs. Motivados por el movimento olımpico, en escuelas ubicadas a lo largo de

todo el territorio nacional, se han desarrollado innumerables talleres de resolucion de

problemas, donde estudiantes y profesores trabajan con el unico afan de incrementar

sus capacidades para el razonamiento, el analisis y la creatividad matematica.

En el ambito internacional, mediante la destacada participacion de las delegaciones

mexicanas en diversos concursos, la Olimpiada Mexicana de Matematicas ha contri-

buido a elevar el prestigio de la matematica nacional. Pero, mas importante aun ha sido

la contribucion que el movimiento olımpico ha tenido para el desarrollo cientıfico del

paıs. En muchos casos, la deteccion temprana de jovenes con talento matematico ex-

cepcional ha permitido brindarles una formacion adecuada para desarrollar al maximo

todo su potencial. Asimismo, la participacion en los concursos olımpicos ha definido

las vocaciones de muchos otros estudiantes. Universidades de todo el paıs se han vis-

to beneficiadas con el ingreso de jovenes ex-olımpicos, mismos que cuentan con una

solida formacion matematica y muchos de los cuales han permanecido en ellas para

dedicar su vida profesional a la docencia y la investigacion.

33a Olimpiada Mexicana de Matematicas

El programa anual de la Olimpiada Mexicana de Matematicas se desarrolla en 3 etapas:

Concursos Estatales.

Concurso Nacional.

Entrenamiento, seleccion y participacion de las delgaciones nacionales que re-

presentan a Mexico en concursos internacionales.

En la 33a Olimpiada Mexicana de Matematicas podran participar los estudiantes de

Mexico nacidos despues del 1 de agosto de 2000. Los concursantes deberan estar ins-

critos en una institucion preuniversitaria durante el primer semestre del ciclo escolar

2019-2020 y, para el 1 de julio de 2020, no deberan haber iniciado estudios universi-

tarios. Para mayor informacion puedes consultar la pagina:

http://www.ommenlinea.org.

Page 6: TZALOA Revista de la Olimpiada Mexicana de Matematicas ... · de un tiempo, Sissa, le presento´ a su rey el juego que le hab´ıa pedido: el ajedrez. Lue-go de entender el juego

VI Presentacion

Para la primera etapa, los participantes deberan inscribirse directamente con el Comite

Estatal correspondiente.

El Concurso Nacional de la 33a Olimpiada Mexicana de Matematicas se realizara del

10 al 15 de noviembre de 2019 en la Ciudad de Mexico. A los primeros lugares de este

certamen se les invitara a la etapa de entrenamiento y seleccion que se realizara durante

aproximadamente diez dıas de cada seis semanas a partir de diciembre de 2019 y hasta

la fecha de celebracion del concurso internacional correspondiente.

Los alumnos que continuen en los entrenamientos nacionales en el mes de marzo, pre-

sentaran el examen de la XXXII Olimpiada de la Cuenca del Pacıfico.

Con base en el desempeno de los participantes durante ese periodo, se elegira a los

integrantes de las delegaciones mexicanas que asistiran a la 61a Olimpiada Internacio-

nal de Matematicas (Rusia, julio de 2020) y a la XXXV Olimpiada Iberoamericana de

Matematicas (Peru, septiembre de 2020).

De entre los concursantes nacidos en 2003 o despues y premiados en el Concurso Na-

cional se seleccionara la delegacion que representara a Mexico en la XXII Olimpiada

Matematica de Centroamerica y el Caribe (Panama, junio de 2020).

De entre las mujeres participantes se seleccionara a la delegacion que representara a

Mexico en la IX Olimpiada Europea Femenil de Matematicas (EGMO) a celebrarse en

el mes de abril de 2020.

3a Olimpiada Mexicana de Matematicas para Educacion Basica

En el ano 2019, la Olimpiada Mexicana de Matematicas (OMM) organiza la Tercera

Olimpiada Mexicana de Matematicas para Educacion Basica (OMMEB). Podran par-

ticipar los alumnos de Primaria y Secundaria, de acuerdo a los siguientes niveles.

Nivel I. Estudiantes de cuarto y quinto ano de primaria o una institucion equivalente.

Los estudiantes no deben haber cumplido 12 anos al 1 de julio de 2019.

Nivel II. Estudiantes de sexto ano de primaria y primer ano de secundaria o una insti-

tucion equivalente. Los estudiantes no deben haber cumplido 14 anos al 1 de julio de

2019.

Nivel III. Estudiantes de segundo ano de secundaria o una institucion equivalente. Los

estudiantes no deben haber cumplido 15 anos al 1 de julio de 2019.

La participacion en la competencia es a traves de los comites estatales de la OMMEB.

El concurso nacional de la 3a OMMEB se realizara en Oaxtepec, Morelos, del 14 al 17

de junio de 2019. Cada Estado participante lo puede hacer con a lo mas un equipo en

cada categorıa. Cada equipo estara integrado por un maximo de 4 personas: un lıder y

3 estudiantes (una misma persona puede ser lıder de mas de un equipo).

Page 7: TZALOA Revista de la Olimpiada Mexicana de Matematicas ... · de un tiempo, Sissa, le presento´ a su rey el juego que le hab´ıa pedido: el ajedrez. Lue-go de entender el juego

Presentacion VII

Habra dos tipos de examenes: individual y por equipos. La prueba individual para el

nivel I constara de 15 problemas a responder en 90 minutos, mientras que para los

niveles II y III, constara de dos partes. La parte A consistira de 12 problemas en la cual

solo la respuesta es requerida. La parte B consistira de 3 problemas y las soluciones

tendran que ir acompanadas de argumentos o explicaciones que sustenten la respuesta.

La prueba por equipos en los tres niveles, consistira de 8 problemas a resolver en 60

minutos.

Los ganadores de los distintos niveles se preseleccionaran para recibir entrenamien-

to y presentar examenes selectivos para conformar a los equipos que representaran a

Mexico en la Competencia Internacional de Matematicas (IMC), que se celebrara en el

verano de 2020.

Page 8: TZALOA Revista de la Olimpiada Mexicana de Matematicas ... · de un tiempo, Sissa, le presento´ a su rey el juego que le hab´ıa pedido: el ajedrez. Lue-go de entender el juego

VIII Presentacion

Page 9: TZALOA Revista de la Olimpiada Mexicana de Matematicas ... · de un tiempo, Sissa, le presento´ a su rey el juego que le hab´ıa pedido: el ajedrez. Lue-go de entender el juego

Ajedrez y Matematicas

Por Victor Manuel Grijalva Altamirano

Nivel Intermedio

En la olimpiada de matematicas, es muy comun encontrar problemas que involucren

un tablero de ajedrez. Dicho tablero, puede ser el clasico de 8 × 8 o de n ×m, donde

n y m son numeros naturales. Es mucha la variedad de este tipo problemas, que para

resolverlos necesitamos saber distintas tecnicas de combinatoria tales como: Principio

de casillas, Invarianza, Coloreado, Teorıa de juegos, Teorıa de grafos, entre otros. En

este artıculo, abordaremos una seleccion de ejemplos resueltos con el objetivo de que

el lector adquiera familiaridad con este tipo de problemas y ponga en practica muchas

de las tecnicas previamente mencionadas.

Un posible origen

El origen del ajedrez es muy incierto, y hasta el dıa de hoy no hay una version oficial.

Muchos historiadores opinan que el ajedrez nacio en la india en el siglo V I , y ademas,

de que el ajedrez proviene de otro juego llamado Chaturanga. Aunque hay una leyenda

muy popular sobre su origen: La leyenda del ajedrez. Cuenta la leyenda que hace mu-

cho tiempo en un reino de la india gobernaba un rey llamado Sheram, un dıa, ordeno

a uno de sus sirvientes, Sissa, que creara un juego que consiguiera divertirle. Despues

de un tiempo, Sissa, le presento a su rey el juego que le habıa pedido: el ajedrez. Lue-

go de entender el juego y jugar varias veces, el rey Sheram quedo sorprendido ante el

maravilloso juego, ası que en agradecimiento le djo a Sissa que como recompensa le

pidiera lo que deseara. Despues de la insistencia del rey, Sissa dijo lo siguiente:

¡Oh, gran rey! Dame tantos granos de trigo como quepan en las 64 casillas del tablero

de ajedrez, de tal manera que se ponga un grano en la primera casilla, dos en la

segunda, cuatro en la tercera, ocho para la cuarta y que siga duplicandose hasta la

casilla 64.

Page 10: TZALOA Revista de la Olimpiada Mexicana de Matematicas ... · de un tiempo, Sissa, le presento´ a su rey el juego que le hab´ıa pedido: el ajedrez. Lue-go de entender el juego

2 V. M. Grijalva Altamirano

El rey sorprendido por la “modesta” peticion de Sissa acepto y mando a los matemati-

cos de la corte a que calcularan la cantidad de granos de trigo que Sissa habıa pedido,

es decir: 1 + 2+ 22 + · · ·+ 263. Despues de que sus sirvientes hicieron las cuentas, el

rey Sheram se dio cuenta de que ni con todo el trigo de su reino alcanzarıa dicha cifra:

18,446,744,073,709,551,615 granos de trigo. El rey Sheram sonrio, pues la peticion de

Sissa era todo menos “modesta”.

Quizas nunca conozcamos el verdadero origen del ajedrez, lo que sı es un hecho es

que ha causado una gran pasion desde su invencion, pues el ajedrez es: Un juego, un

deporte y un arte.

Ajedrez y matematicos

Grandes matematicos como George Polya, Carl Gauss, L. Euler, Donald E. Knuth, Le-

gendre, etc., se interesaron por problemas matematicos en el ajedrez. A continuacion,

mencionamos algunos de esos problemas.

El problema de las ocho damas

El problema de las ocho damas es un pasatiempo que consiste en contar todas las ma-

neras posibles de poner 8 damas en el tablero de ajedrez de 8×8 sin que ninguna dama

amenace a otra. En la actualidad se sabe que hay 92 maneras de hacer dicha tarea, pero

por mucho tiempo fue un reto que cautivo a muchas personas.

Fue propuesto por primera vez por el ajedrecista aleman Maxx Bezzel en septiembre

de 1848 en el periodico de ajedrez Schachzeitung. Dos soluciones fueron publicadas

en enero de 1849 y un total de 40 soluciones aparecieron en Schachzeitung entre los

anos 1849 y 1854. El problema fue otra vez propuesto por Franz Nauck el 1 de junio de

1850 en el periodico Illustrirte Zeitung, este periodico contaba con mayor numero de

lectores y entre ellos el matematico Gauss. Nauck tambien considero una variante del

problema: encontrar todas las soluciones con damas colocadas en las casillas b4 y d5. El

29 junio de 1850, Nauck afirmo en Illustrirte Zeitung, que el problema principal tenıa

60 soluciones. Unos meses despues, el 21 de septiembre, Nauck hace una correccion y

da las 92 soluciones en Illustrirte Zeitung.

El problema del caballo

El problema del caballo es un problema antiguo, el cual consiste que dado un tablero

de ajedrez de n×n casillas y un caballo de ajedrez colocado en alguna de las casillas, el

caballo pase por todas las casillas una sola vez siguiendo el movimiento usual del caba-

llo. Se han encontrado muchas soluciones a este problema; a continuacion mostramos

la solucion del matematico Leonhard Euler.

Page 11: TZALOA Revista de la Olimpiada Mexicana de Matematicas ... · de un tiempo, Sissa, le presento´ a su rey el juego que le hab´ıa pedido: el ajedrez. Lue-go de entender el juego

Ajedrez y Matematicas 3

1 48 31 50 33 16 63 18

30 51 46 3 62 19 14 35

47 2 49 32 15 34 17 64

52 29 4 45 20 61 36 13

5 44 25 56 9 40 21 60

28 53 8 41 24 57 12 37

43 6 55 26 39 10 59 22

54 27 42 7 58 23 38 11

En el tablero anterior, cada casilla tiene un numero, dicho numero indica el orden del

movimiento que debe seguir el caballo para recorrer el tablero. El problema del caballo

tiene algunas variaciones, tales como:

1) Buscar soluciones, en las cuales se debe llegar a la misma casilla de la cual se partio.

2) Tableros de diferente numero de columnas o diferente numero de filas.

Ejemplos resueltos

Problema 1. Considere los dos tableros de ajedrez de 3 × 3 que se muestran a conti-

nuacion, en cada uno de ellos hay dos caballos blancos y dos caballos negros. Movien-

do una pieza a la vez tantas veces como queramos y siguiendo el movimiento estandar

del caballo en el juego de ajedrez, ¿es posible mover las piezas del tablero izquierdo

de tal forma que nos quede el tablero derecho? (Nota: No puede haber dos caballos en

un mismo cuadrito).

3 M0m2 0Z01 M0m

a b c

3 m0M2 0Z01 M0m

a b c

Solucion. Resolveremos este problema con ayuda de un grafo. A cada casilla del ta-

blero le asignaremos una letra como muestra el siguiente diagrama:

a b c

d e f

g h i

Page 12: TZALOA Revista de la Olimpiada Mexicana de Matematicas ... · de un tiempo, Sissa, le presento´ a su rey el juego que le hab´ıa pedido: el ajedrez. Lue-go de entender el juego

4 V. M. Grijalva Altamirano

Cada una de estas letras formaran los vertices de un grafo. Los vertices X y Y seran

unidos con una arista, si es posible moverse con el caballo de la casilla marcada con la

letra X a la casilla marcada con la letra Y en un movimiento. Si aplicamos esta regla a

cada una de las casillas obtenemos el siguiente grafo:

a

h

f

e

c

d

i

bg

b

b b

b

b

b

b

bb

Ahora le asignaremos un grafo a cada uno de los dos tableros, agregaremos la etiqueta

N a los vertices en donde se encuentre un caballo negro y B a los vertices en donde se

encuentre un caballo blanco. Quedando ası, los siguientes dos grafos:

a

h

f

e

c

d

i

bg

B

B

N

N

b

b b

b

b

b

b

bb

a

h

f

e

c

d

i

bg

B

B

N

N

b

b b

b

b

b

b

bb

Comparando ambos grafos podemos observar que el orden de las piezas negras y blan-

cas son diferentes. De lo cual podemos concluir que no es posible cambiar el orden de

los caballos, pues un caballo solo puede moverse sobre los vertices vacıos. De aquı, es

imposible mover las piezas del tablero izquierdo de tal forma que nos quede el tablero

derecho.

Problema 2. Sobre un tablero de ajedrez de 2020× 2018, A y B juegan por turnos a

mover un caballo sobre el tablero (movimiento en forma de L). Inicialmente, el tablero

no tiene ninguna pieza. En un principio, el jugador A coloca una pieza del caballo en

cualquiera de las 4076360 casillas libres. Luego, le toca el turno al jugador B, quien

solo puede mover el caballo a una nueva casilla no visitada antes. Despues, le toca

mover al jugador A, el cual debe mover el caballo a otra casilla no visitada antes en

ninguno de los turnos anteriores, y ası, sucesivamente. En este juego, pierde el primer

jugador que no pueda mover el caballo a una nueva casilla. Determinar que jugador

tiene estrategia ganadora.

Solucion. Mostraremos que el jugador B tiene estrategia ganadora, es decir, sin impor-

tar el movimiento que realice el jugador A, el jugador B ganara este juego. Para ello,

observemos el comportamiento del caballo sobre un tablero de 4× 2. En un tablero de

4× 2, enumeramos las 4 casillas de la primera columna con los numeros 1, 2, 3 y 4. Si

colocamos un caballo en la casilla con el numero 1 y realizamos un movimiento legal

Page 13: TZALOA Revista de la Olimpiada Mexicana de Matematicas ... · de un tiempo, Sissa, le presento´ a su rey el juego que le hab´ıa pedido: el ajedrez. Lue-go de entender el juego

Ajedrez y Matematicas 5

del caballo, en el mismo tablero de 4 × 2 encontraremos otra casilla al que podemos

llegar. Esta nueva casilla lo enumeramos con el mismo numero 1. Este mismo procedi-

miento lo haremos para los tres numeros restantes. El tablero quedara como el que se

muestra a continuacion:1 3

2 4

3 1

4 2

Observemos que en cada tablero de 4 × 2, a cada casilla X le corresponde una unica

casilla Y a la cual podemos llegar moviendo el caballo apartir de X . Ahora dividamos

el tablero de ajedrez 2020 × 2018, en tableros de 4 × 2 (nos quedaran exactamente

509545 tableros de 4 × 2 y no sobra ninguna casilla). Procederemos a probar que el

jugador B tiene la estrategia ganadora. Al inicio el jugador A coloca el caballo en una

casilla al azar. Sin importar cual elige A, coloco el caballo en una casilla de algun

tablero de 4 × 2. La estrategıa del jugador B, es mover el caballo a la otra casilla a la

cual se puede llegar y que esta en el mismo tablero de 4 × 2. Esto obligara al jugador

A a buscar otro movimiento en otro tablero de 4× 2 y nuevamente B mueve el caballo

a la otra casilla a la cual se puede llegar y que esta en el mismo tablero de 4 × 2. Si

continua con esta estrategia el jugador B, el jugador A tiene dos posibilidades.

Caso 1. En algun momento el jugador A no puede mover a una casilla no visitada.

Caso 2. El jugador A juega de forma muy acertada y realiza el movimiento numero

4076359 (el penultimo movimiento del juego). Luego, el jugador B solo tiene que

mover el caballo a la otra casilla a la cual se puede llegar y que esta en el mismo

tablero de 4× 2. Ası, el jugador B realiza el ultimo movimiento del juego.

En ambos casos el jugador A pierde, por lo cual el jugador B gana el juego.

En muchos problemas de combinatoria es muy util aplicar el Principio de las Casillas,

a continuacion enunciamos dicho principio:

Proposicion 1 (Principio de las Casillas). Si n objetos son acomodados en k lugares,

entonces hay al menos ⌈nk ⌉ objetos en el mismo lugar.

Problema 3. (1a Olimpiada Matematica de Colorado, 1984) En un tablero de ajedrez

de 10×10, se han colocado 41 torres. Pruebe que entre esas 41 torres hay 5 torres que

no se atacan mutuamente (es decir, no hay dos de ellas en la misma columna o fila).

Solucion. En este problema, necesitamos encontrar 5 torres que no esten colocadas en

la misma fila y no esten colocadas en la misma columna. El uso adecuado del principio

de las casillas en este problema nos garantizara la existencia de las 5 torres deseadas.

Observemos que los objetos a acomodar son las 41 torres, y deseamos acomodarlas en

10 filas. Luego, por el principio de las casillas, hay una fila con al menos ⌈ 4110⌉ = 5torres. Hasta aquı, no hemos resuelto el problema, pues las 5 torres se encuentran en

la misma fila. Necesitamos seguir trabajando con las filas restantes. Notemos que cada

fila puede tener a lo mas 10 torres, ası, si eliminamos la fila en donde se encuentran

las 5 torres, nos quedaran 9 filas y como mınimo 31 torres. Ahora, deseamos acomodar

Page 14: TZALOA Revista de la Olimpiada Mexicana de Matematicas ... · de un tiempo, Sissa, le presento´ a su rey el juego que le hab´ıa pedido: el ajedrez. Lue-go de entender el juego

6 V. M. Grijalva Altamirano

31 torres en 9 filas; por el principio de las casillas, se sigue que hay una fila con al

menos ⌈ 319 ⌉ = 4 torres. Si eliminamos dicha fila, nos quedaran 8 filas y como mınimo

21 torres. Nuevamente, aplicando el principio de casillas, se tiene que hay una fila con

al menos ⌈ 218 ⌉ = 3 torres. Continuando con este procedimiento, podemos encontrar

que hay 5 filas especiales que tienen al menos 1, 2, 3, 4, 5 torres, respectivamente.

Ahora, busquemos las 5 torres deseadas en estas 5 filas teniendo en cuenta que solo

resta cuidar que se encuentren en distintas columnas. Seleccionemos la torre que se

encuentra en la fila que tiene al menos una torre. Luego, en la fila que tiene al menos dos

torres, seleccionemos una de esas dos torres tal que no este en la misma columna que

la primera torre elegida. Despues, en la fila que tiene al menos 3 torres, seleccionemos

una de esas tres tal que no esta en la misma columna que la primera torre y tampoco en

la misma columna de la segunda torre elegida. Continuando con este procedimiento,

elegiremos la dos torres restantates. Eureka! Hemos encontrado las 5 torres buscadas.

Problema 4. Encuentre todos los enteros positivos n tal que si se remueve una casilla

de un tablero de 2n × 2n, el tablero resultante se puede cubrir completamente usando

las veces que queramos la siguiente figura que llamaremos L.

Solucion. Despues de hacer unos ejemplos sencillos podemos observar que todos los

enteros n ≥ 1 satisfacen el problema. Procederemos a demostrar dicha afirmacion por

induccion en n.

1) Caso base. Consideremos el caso cuando n = 1, ası, el tablero es de 2 × 2. Al

remover una casilla, queda exactamente la misma figura con la que se nos pide

cubrir (salvo rotacion). Luego, solo hacemos uso de una figura L y listo.

2) Paso inductivo. Sea k ≥ 2 un entero, supongamos que podemos cubrir el tablero

de 2k × 2k con una casilla removida. Tenemos que usar de alguna manera nuestra

hipotesis de induccion para probar que podemos cubrir el tablero de 2k+1 × 2k+1

con una casilla removida. Para ello, dividamos el tablero de 2k+1 × 2k+1 en cuatro

subtableros de 2k × 2k, como se muestra en la siguiente figura:

Page 15: TZALOA Revista de la Olimpiada Mexicana de Matematicas ... · de un tiempo, Sissa, le presento´ a su rey el juego que le hab´ıa pedido: el ajedrez. Lue-go de entender el juego

Ajedrez y Matematicas 7

Del tablero de 2k+1×2k+1 eliminaremos una casilla (es la condicion del problema),

dicha casilla tendra que estar en alguno de los cuatro subtableros de 2k × 2k. Para

terminar el problema, eliminemos una casilla en cada uno de los otros tres subta-

bleros de 2k × 2k, de tal manera que formen la figura L, tal como se muestra en la

siguiente figura:

Luego, observemos que cada uno de los cuatro subtableros de 2k × 2k tiene una

casilla removida. Ası, por nuestra hipotesis inductiva, cada uno de los cuatro tablero

se puede cubrir completamente. Recordemos que hemos removido 4 casillas y en el

problema se nos pide remover solo una. Observemos que tres casillas de las cuatro

removidas se pueden cubrir completamente si hacemos uso de solo una figura L.

De aquı, hemos demostrado que es posible cubrir completamente con figuras L un

tablero de 2k+1 × 2k+1 con una casilla removida.

Luego, concluimos por el pricipio de induccion que para cada n ∈ N, todo tablero de

2n × 2n con una casilla removida se puede cubrir completamente con figuras L.

Problema 5. (Concurso Nacional, 26a Olimpiada Mexicana de Matematicas) Algunas

ranas, unas de ellas rojas y otras verdes, se van a mover en un tablero de 11 × 11,

de acuerdo a las siguientes reglas. Si una rana esta ubicada, digamos, en la casilla

marcada con # en la siguiente figura, entonces:

1) Si es roja, puede saltar a cualquiera de las casillas marcadas con X ,

2) Si es verde, puede saltar a cualquiera de las casillas marcadas con O.

O X

#

X O

O X

X O

Diremos que dos ranas (de cualquier color) se pueden encontrar en una casilla si

ambas pueden llegar hasta ella saltando una o mas veces, no necesariamente con el

mismo numero de saltos.

Page 16: TZALOA Revista de la Olimpiada Mexicana de Matematicas ... · de un tiempo, Sissa, le presento´ a su rey el juego que le hab´ıa pedido: el ajedrez. Lue-go de entender el juego

8 V. M. Grijalva Altamirano

a) Muestra que si ponemos 6 ranas (de cualquier color), entonces hay al menos 2 que

se pueden encontrar.

b) ¿Para que valores de k es posible poner una rana roja y una rana verde de manera

que haya exactamente k casillas en las que estas ranas se puedan encontrar?

Solucion. Analicemos primero el movimiento de las ranas rojas sobre el tablero. Para

ello, enumeremos las 5 primeras casillas de la primera fila con los numeros del 1 al

5. Si colocamos una rana roja en la casilla que tiene el numero 1 y realizamos mo-

vimientos permitidos de la rana roja, en el tablero encontraremos varias casillas a las

que podemos llegar, dichas casillas las enumeramos con el numero 1. Repetiremos este

procedimiento para las casillas que tienen los numeros 2,3,4 y 5, respectivamente. El

tablero que resulta de seguir estos pasos es el siguiente:

1 2 3 4 5 1 2 3 4 5 1

3 4 5 1 2 3 4 5 1 2 3

5 1 2 3 4 5 1 2 3 4 5

2 3 4 5 1 2 3 4 5 1 2

4 5 1 2 3 4 5 1 2 3 4

1 2 3 4 5 1 2 3 4 5 1

3 4 5 1 2 3 4 5 1 2 3

5 1 2 3 4 5 1 2 3 4 5

2 3 4 5 1 2 3 4 5 1 2

4 5 1 2 3 4 5 1 2 3 4

1 2 3 4 5 1 2 3 4 5 1

Por la manera en que se construyo el tablero anterior, si una rana roja empieza en la

casilla con el numero X , esta rana puede visitar todas las casillas enumeradas con el

mismo numero X , dondeX ∈ 1, 2, 3, 4, 5. Ademas, una rana puede visitar cualquier

fila y columna del tablero.

Ahora, analicemos la interaccion de las ranas verdes con las ranas rojas. Para ello,

coloquemos una rana verde sobre alguna casilla del tablero que hemos construido pre-

viamente. Por ejemplo, coloquemos una rana verde sobre la casilla superior izquierda

(la casilla que tiene el numero 1). Realicemos movimientos permitidos de la rana ver-

de a partir de dicha casilla. Si marcamos todas las casillas que visitamos, el tablero

quedara como el que se muestra a continuacion:

Page 17: TZALOA Revista de la Olimpiada Mexicana de Matematicas ... · de un tiempo, Sissa, le presento´ a su rey el juego que le hab´ıa pedido: el ajedrez. Lue-go de entender el juego

Ajedrez y Matematicas 9

1 2 3 4 5 1 2 3 4 5 1

3 4 5 1 2 3 4 5 1 2 3

5 1 2 3 4 5 1 2 3 4 5

2 3 4 5 1 2 3 4 5 1 2

4 5 1 2 3 4 5 1 2 3 4

1 2 3 4 5 1 2 3 4 5 1

3 4 5 1 2 3 4 5 1 2 3

5 1 2 3 4 5 1 2 3 4 5

2 3 4 5 1 2 3 4 5 1 2

4 5 1 2 3 4 5 1 2 3 4

1 2 3 4 5 1 2 3 4 5 1

Observemos que una rana verde visita en cada columna y fila solo un tipo de numero.

Notemos que en cada tablero de 5× 5 la rana verde visita los 5 numeros distintos (vea

la figura de arriba). Con estas observaciones, procederemos a demostrar las dos partes

del problema:

Parte (a). Si colocamos 6 ranas en el tablero, puede ocurrir los siguientes dos casos:

Caso 1. En el tablero hay ranas de distintos colores. En este caso, las ranas de distintos

colores se encuentran en cada tablero de 5× 5.

Caso 2. Todas las ranas son del mismo color. Sin perdida de generalidad, supongamos

que todas las ranas son rojas. Ası, por el principio de las casillas, hay al menos ⌈ 62⌉ = 2ranas rojas que al colocarlas en el tablero caen en el mismo numero. Luego, esas dos

ranas se pueden encontrar.

Parte (b). Teniendo en cuenta que en cada tablero de 5 × 5 una rana roja y una verde

se encuentran exactamente una vez, dividiremos el tablero de 11× 11 en 9 regiones: 4tableros de 5× 5 (las 4 regiones comparten un lado en comun), 4 tableros de 5× 1 (las

orillas) y una casilla (la esquina), tal como se muestra a continuacion:

Dado que tenemos 4 tableros de 5 × 5 y ahı ya sabemos que se tienen que encontrar

las dos ranas y ademas, como tenemos 9 regiones en total en todo el tablero se sigue

que 4 ≤ k ≤ 9. Sabemos que en cada tablero de 5 × 5 ambas ranas se tienen que

encontrar, ası que solo nos enfocaremos en el tablero de 5 × 5 que se encuentra en la

parte superior izquierda. Si en ese tablero de 5 × 5 colocamos ambas ranas sobre la

casilla superior izquierda (el que tiene el numero 1), ambas ranas se encontraran en las

9 regiones en las que hemos dividido el tablero de 11× 11. Si colocamos ambas ranas

Page 18: TZALOA Revista de la Olimpiada Mexicana de Matematicas ... · de un tiempo, Sissa, le presento´ a su rey el juego que le hab´ıa pedido: el ajedrez. Lue-go de entender el juego

10 V. M. Grijalva Altamirano

en otra casilla del tablero de 5×5 que no sea la que ya hemos analizado, solo podremos

visitar 4 o 6 regiones. Esto se debe a que no siempre es posible visitar las 5 regiones

que se encuentran en la orilla (vea la figura de arriba). Por lo tanto, k = 4, 6 o 9.

Problema 6. (Olimpiada Matematica Rusa, 2000) Considere un tablero de 200×200.

Cada casilla del tablero es coloreado de color blanco o negro, de tal manera que el

numero de casillas negras menos el numero de casillas blancas en el tablero es 404.

Pruebe que sin importar como se coloree el tablero siempre y cuando se cumpla con

la condicion mencionada, existe un cuadrado de 2× 2 sobre el tablero tal que tiene un

numero impar de casillas negras.

Solucion. Procederemos por contradiccion. Supongamos que todos los cuadrados de

2 × 2 sobre el tablero tienen un numero par de casillas negras. Denotaremos por B y

N al numero de casillas blancas y casillas negras de la primera columna, respectiva-

mente. Observemos que B +N = 200. Como estamos bajo el supuesto que todos los

cuadrados de 2 × 2 sobre el tablero tienen un numero par de casillas negras, notemos

que una vez coloreado la primera columna, la forma de colorear la segunda columna

solo tiene dos posibilidades:

1) Lo pintamos de la misma manera en que pintamos la primera columna.

2) Lo pintamos de la forma opuesta en que hemos pintado la primera columna.

De forma similar, una vez coloreado la segunda columna, la tercera columna solo tiene

dos formas de colorearse. En general, la forma en que coloreemos la primera columna

nos dice las dos opciones que tienen las demas columnas de colorearse. El siguiente

ejemplo, muestra un tablero de 5× 5. En el tablero, la segunda columna esta coloreada

en forma opuesta a la manera que se coloreo la primera columna. La tercera colum-

na esta coloreado de la misma manera en que se coloreo la segunda columna, y ası,

sucesivamente.

Retomando nuestro problema, denotaremos por x al numero de columnas del tablero

coloreado exactamente en la misma manera que la primera columna, y denotaremos

por y al numero de columnas del tablero coloreado exactamente de la forma opuesta

al de la primera columna. Observemos que x + y = 200. Notemos que Nx + Byes igual al numero casillas negras en todo el tablero y Bx + Ny es igual al numero

casillas blancas en todo el tablero. Por las condiciones del problema, tenemos que

(Nx + By) − (Bx + Ny) = 404. Haciendo el uso adecuado del algebra deducimos

que (x − y)(N −B) = 404. Por otro lado, dado que x + y = 200, se sigue que x y y

Page 19: TZALOA Revista de la Olimpiada Mexicana de Matematicas ... · de un tiempo, Sissa, le presento´ a su rey el juego que le hab´ıa pedido: el ajedrez. Lue-go de entender el juego

Ajedrez y Matematicas 11

tienen la misma paridad (par + par= par e impar + impar=par). De forma similar, B y

N tienen la misma paridad. De aquı, x − y = 2m y B − N = 2n, para algun entero

m y n. Observemos que 1 ≤ x ≤ 200 y 0 ≤ y ≤ 199, luego −200 ≤ x − y = 2m ≤200, es decir, |m| ≤ 100. De forma analoga, se tiene que |n| ≤ 100. Por otro lado,

(x − y)(N − B) = 4nm = 404. Ası, mn = 101. Lo cual es una contradiccion, pues

101 es primo y ademas |m| ≤ 100 y |n| ≤ 100.

Problema 7. (Lista corta, Olimpiada Internacional, 2002) Dado un entero positivo n,

un tablero de (2n+1)× (2n+1) sera cubierto por piezas de los tipos que se muestran

a continuacion:

donde rotaciones y reflexiones de las piezas son permitidas. Pruebe que por lo menos

4n+ 3 piezas del primer tipo seran usadas.

Solucion. Dado el tablero de (2n+1)× (2n+1), numeremos las filas y columnas del

1 al 2n + 1. Las casillas que se encuentren en una fila impar y columna impar se van

a colorear de negro y las otra casillas se dejaran en blanco. Por ejemplo, para el caso

n = 3, tenemos un tablero de 7×7 y queda coloreado como se muestra a continuacion:

Si coloreamos el tablero de (2n + 1) × (2n + 1) como se ha indicado, dicho tablero

tendra (n+1)2 = n2+2n+1 casillas negras y 3n2+2n casillas blancas (al numero total

de casillas del tablero restele el numero de casillas negras). Al colocar una pieza del

primer tipo, notemos que esta pieza cubre dos casillas blancas y una negra, o cubre tres

casillas blancas. Las otras dos piezas, siempre cubren 3 casillas blancas y una negra.

Sea A el numero de piezas del primer tipo que cubre dos casillas blancas y una negra,

B el numero de piezas del primer tipo que cubre tres casillas blancas y C el numero

de piezas de los otros dos tipos. Contando las casillas negras, tenemos que A + C =n2 + 2n+ 1. Contando las casillas blancas, tenemos que 2A+ 3B +3C = 3n2 + 2n.

Ahora, notemos que:

4n+ 3 = 3(n2 + 2n+ 1)− (3n2 + 2n) = 3(A+ C)− (2A+ 3B + 3C)

= A− 3B ≤ A+B.

Page 20: TZALOA Revista de la Olimpiada Mexicana de Matematicas ... · de un tiempo, Sissa, le presento´ a su rey el juego que le hab´ıa pedido: el ajedrez. Lue-go de entender el juego

12 V. M. Grijalva Altamirano

En consecuencia, para cubrir el tablero necesitamos al menos 4n+3 piezas del primer

tipo.

Problema 8. Considere un tablero de 2×2n. Encuentre el numero total de maneras de

colocar n piezas del rey en el tablero tal que estos no se ataquen mutuamente (un rey

ataca a una pieza si este se encuentra en una casilla adyacente). El siguiente diagrama,

muestra un ejemplo de 4 piezas de rey que no se atacan mutuamente en un tablero de

2× 8 (la letra R indica que se ha colocado una pieza del Rey en dicha casilla).

R

R R

R

Solucion. Este problema lo resolveremos por recursion. Denotaremos por tn al numero

total de maneras de colocar n piezas del rey en el tablero de 2× 2n tal que estos no se

ataquen mutuamente. Empezaremos analizando los casos cuando n = 1 y n = 2.

R

R

Cuando n = 1, podemos notar que solo hay 4 posibilidades para colocar a un rey en un

tablero de 2 × 2, ası, t1 = 4. Lo interesante del problema surge cuando analizamos el

caso cuando n = 2. Debemos tener cuidado al colocar las piezas del rey, de no hacerlo,

puede ocurrir el caso que muestra el diagrama de arriba (el rey de la columna 2 ataca

al rey de la columna 3). Despues de intentar muchos casos, podemos convencernos que

t2 = 12. Por la naturaleza del problema, nos conviene dividir el tablero de 2 × 2n en

n cuadrados de 2 × 2. Estudiaremos el caso cuando n = 3 y de ahı generalizaremos.

Recordemos que hemos dividido el tablero de 2 × 6 en 3 cuadrados de 2 × 2 (vea el

diagrama de abajo). El primero cuadrado de 2× 2 es el que tiene las letras: a, b, c y d.

El segundo cuadrado de 2× 2 es el que tiene las letras: e, f , g y h. El tercer cuadrado

de 2 × 2 es el que tiene las letras: i, j, k y l. Hallaremos el valor de t3 teniendo en

cuenta que t1 = 4 nos dice el numero de formas de colocar a una pieza del rey en un

tablero de 2 × 2 y t2 = 12 nos dice el numero de formas de colocar a dos piezas del

rey en un tablero de 2× 4.

a

b

c

d

e

f

g

h

i

j

k

l

Fijemos nuestra atencion en el tercer cuadrado de 2 × 2. Dado que ya conocemos la

solucion para el tablero de 2× 4 (el valor de t2) y como en el tercer cuadrado de 2× 2podemos colocar a una pieza del rey en cualquiera de sus 4 casillas, nuestra primera

estimacion para t3 serıa t3 = 4t2. Pero como mencionamos previamente, hay que tener

cuidado al colocar una pieza del rey en la segunda columna del segundo cuadrado (g

Page 21: TZALOA Revista de la Olimpiada Mexicana de Matematicas ... · de un tiempo, Sissa, le presento´ a su rey el juego que le hab´ıa pedido: el ajedrez. Lue-go de entender el juego

Ajedrez y Matematicas 13

y h) y al colocar una pieza del rey en la primera columna del tercer cuadrado (i y j).

Hay 4 maneras de colocar al mismo tiempo las dos piezas del rey en las dos columnas

que no queremos y, cada una de esas maneras, dejan fijo a un rey en el segundo y en el

tercer cuadrado de 2×2. Ası, en cada uno de esos 4 casos, estamos agregando a nuestra

primera estimacion de t3, la solucion del primer cuadrado de 2 × 2, esto es, t1. Para

eliminar aquellos casos que no son de nuestro interes (el que acabamos de mencionar)

basta con notar que t3 = 4t2−4t1. En general, para n ≥ 3, tn = 4tn−1−4tn−2 nos da

el numero total de maneras de colocar n piezas del rey en el tablero de 2 × 2n tal que

estos no se ataquen mutuamente. Ahora, procederemos a obtener una formula cerrada

para tn. Como tn = 4tn−1 − 4tn−2, se sigue que:

tn − 2tn−1 = 2(tn−1 − 2tn−2) = 2(2(tn−2 − 2tn−3)) = · · · = 2n−2(t2 − 2t1) = 2n.

De aquı, tn − 2tn−1 = 2n, lo cual implica que

tn − 2tn−1 = 2n

2tn−1 − 4tn−2 = 2n

4tn−2 − 8tn−3 = 2n

...

2n−2t2 − 2n−1t1 = 2n.

Sumando todos los terminos del lado izquierdo y del lado derecho, se sigue que tn −2n−1t1 = 2n(n− 1). De lo cual deducimos que tn = 2n(n+ 1).

Ahora, dejamos unos ejercicios para el lector.

Ejercicios

1) Sea k un entero no negativo. ¿Para que enteros positivos n de la forma 6k + 3 es

posible cubrir un tablero de n × n completamente, usando solamente piezas como

la siguiente figura?

2) Sobre un tablero de 8 × 8, A y B juegan por turnos a colocar caballos negros y

caballos blancos, respectivamente. Un jugador pierde si el coloca un caballo sobre

una casilla el cual se encuentra atacado por algun caballo de su oponente, o si no ya

no hay casillas libres para colocar su caballo. Si el jugador A empieza, ¿quien tiene

la estrategia ganadora?

3) Determine el numero de maneras de cubrir un tablero de 4 × 4 usando 8 fichas de

domino (2× 1).

4) En cada cuadrado de una cuadrıcula de 6×6 hay una luciernaga apagada o encendi-

da. Una movida es escoger tres cuadrados consecutivos, ya sean los tres verticales o

Page 22: TZALOA Revista de la Olimpiada Mexicana de Matematicas ... · de un tiempo, Sissa, le presento´ a su rey el juego que le hab´ıa pedido: el ajedrez. Lue-go de entender el juego

14 V. M. Grijalva Altamirano

los tres horizontales, y cambiar de estado a las tres luciernagas que se encuentran en

dichos cuadrados. Cambiar de estado a una luciernaga significa que si esta apagada

se enciende y viceversa. Muestre que si inicialmente hay una luciernaga encendida

y las demas apagadas, no es posible hacer una serie de movidas tales que al final

todas las luciernagas esten apagadas.

5) Dado un tablero de 8 × 8, ¿de cuantas formas es posible colocar en cada casilla el

numero −1 o el numero 1, de tal forma que la suma de cada tablero de 2 × 2 sea

igual a 0?

6) Sea n un numero entero mayor que 1. ¿De cuantas formas se pueden acomodar

todos los numeros 1, 2, 3, . . . , 2n en las casillas de una cuadrıcula de 2× n, uno en

cada casilla, de manera que cualesquiera dos numeros consecutivos se encuentren

en casillas que comparten un lado en la cuadrıcula?

7) Los enteros del 1 al n2(n ≥ 2) son colocadas arbitrariamente sobre un tablero de

n× n. Pruebe que existen dos casillas adyacentes (comparten un vertice en comun

o un lado en comun) tal que la diferencia de los numeros colocadas en ellas no es

menor que n+ 1.

8) Considere un tablero de 4×1995. ¿Puede un caballo moverse sobre todas las casillas

del tablero (puede visitar una casilla solo una vez) de tal forma que la ultima casilla

de su recorrido sea la primera en donde empezo?

9) En un tablero de ajedrez de 2017 × 2017, se han colocado en la primera columna

2017 caballos de ajedrez, uno en cada casilla de la columna. Una tirada consiste

en elegir dos caballos distintos y de manera simultanea moverlos como se mueven

los caballos de ajedrez. Encuentra todos los posibles valores enteros de k con 1 ≤k ≤ 2017, para los cuales es posible llegar a traves de varias tiradas, a que todos los

caballos esten en la columna k, uno en cada casilla.

10) En cada casilla de un tablero de n × n hay un foco. Inicialmente todos los focos

estan apagados. En un paso, se permite cambiar el estado de todos los focos en una

fila o de todos los focos en una columna (los focos prendidos se apagan y los focos

apagados se prenden). Muestra que si despues de cierta cantidad de pasos hay uno

o mas focos prendidos, entonces en ese momento hay al menos n focos prendidos.

Bibliografıa

1) Alexander Soifer. The Colorado Mathematical Olympiad: The Third Decade and

Further Explorations. Springer, 2011.

2) Bin Xiong, Zhongyi Zheng. Graph Theory. Mathematical Olympiad Series, Vol. 3.

East China Normal University Press, 2010.

3) Vlad Matei, Elizabeth Reiland. 112 Combinatorial Problems from the Awesome-

Math Summer Program. Vol. 21, XYZ Press, 2016.

4) Yao Zhang. Combinatorial problems in mathematical competitions. Vol. 4, Mathe-

matical Olympiad Series, 2011.

Page 23: TZALOA Revista de la Olimpiada Mexicana de Matematicas ... · de un tiempo, Sissa, le presento´ a su rey el juego que le hab´ıa pedido: el ajedrez. Lue-go de entender el juego

Problemas de practica

A continuacion presentamos los problemas del examen de invitacion a la OMM del ano

2018. Los problemas del 1 al 12 conformaron la version A del examen y los problemas

del 13 al 25 conformaron la version B del examen.

Problema 1. ¿Que construccion no puede hacerse usando las dos piezas que se mues-

tran?

(b)(a) (c) (e)(d)

Problema 2. Jacobo quiere insertar el dıgito 3 en el numero 2018 de manera que el

numero de 5 dıgitos que forme sea lo mas pequeno posible. ¿Donde debe colocarlo?

(a) antes del 2 (b) entre el 2 y el 0 (c) entre el 0 y el 1(d) entre el 1 y el 8 (e) despues del 8

Problema 3. Sobre cada lado de un cuadrado de lado 20 cm, se coloca en su exterior

un cuadrado de lado 5 cm.

20 cm

5 cm

Page 24: TZALOA Revista de la Olimpiada Mexicana de Matematicas ... · de un tiempo, Sissa, le presento´ a su rey el juego que le hab´ıa pedido: el ajedrez. Lue-go de entender el juego

16 Problemas de practica

¿Cual es el perımetro de la figura que se formo?

(a) 80 cm (b) 100 cm (c) 110 cm (d) 120 cm (e) 140 cm

Problema 4. Las caras de un cubo estan pintadas con tres colores de manera que caras

opuestas son del mismo color. ¿Cual de las siguientes figuras corresponde al desarrollo

del cubo?

(a) (b) (c) (d) (e)

Problema 5. En la figura se muestran entrelazados un anillo gris y uno blanco. Pedro,

que esta enfrente de los anillos, los ve como se muestra en la figura.

¿Como los ve Pablo si esta detras de los anillos?

(a) (b) (c) (d) (e)

Problema 6. Pancho hizo una hilera con 7 fichas de domino de manera que los lados

con el mismo numero de puntos quedaran uno al lado del otro. Originalmente la hilera

tenıa un total de 33 puntos, pero el hermanito de Pancho se llevo dos de las fichas.

¿Que cantidad de puntos habıa en el lugar que senala la flecha en la figura?

b bb

b b

b b

b b

bb

b

bb

bbb

bb

b

b

b

(a) 2 (b) 3 (c) 4 (d) 5 (e) 6

Problema 7. Flora construyo el paralelepıpedo que se muestra en la figura usando 3piezas de distintos colores, de 4 cubitos cada una. En el dibujo se ven los cuatro cubitos

de dos de las piezas; de la tercera pieza se ven solo 2 de los 4 cubitos. ¿Que forma tiene

la tercera pieza?

Page 25: TZALOA Revista de la Olimpiada Mexicana de Matematicas ... · de un tiempo, Sissa, le presento´ a su rey el juego que le hab´ıa pedido: el ajedrez. Lue-go de entender el juego

Problemas de practica 17

(a) (b) (c) (d) (e)

Problema 8. Pablo y Emilio tomaron agua de una jarra que estaba llena. Emilio tomo

primero cierta cantidad, Pablo tomo despues una cuarta parte de lo que quedaba. Si los

dos tomaron la mitad del agua de la jarra, ¿que fraccion de la jarra tomo Pablo?

(a) 18 (b) 1

6 (c) 14 (d) 1

3 (e) 12

Problema 9. ¿Cual es el menor numero de piezas en forma de L, como la de la figura,

que se necesitan para formar un tablero cuadrado? (Las piezas pueden girarse, pero no

se pueden recortar).

(a) 5 (b) 10 (c) 15 (d) 20 (e) 25

Problema 10. Ana, Bebe y Cici se deben de sentar en alguna de 7 sillas que estan en

fila, de manera que entre cada dos de ellas quede al menos una silla vacıa. ¿De cuantas

maneras se puede hacer este acomodo?

(a) 6 (b) 7 (c) 36 (d) 42 (e) 60

Problema 11. Un cubo que mide 2×2×2 esta formado por cuatro cubos transparentes

de 1×1×1 y cuatro cubos opacos (no transparentes) de 1×1×1 como se muestra en la

figura. Estan colocados de tal manera que el cubo grande completo no es transparente

(es decir, no es posible ver de adelante hacia atras, ni de arriba hacia abajo, ni de lado

a lado).

Page 26: TZALOA Revista de la Olimpiada Mexicana de Matematicas ... · de un tiempo, Sissa, le presento´ a su rey el juego que le hab´ıa pedido: el ajedrez. Lue-go de entender el juego

18 Problemas de practica

Al menos, ¿cuantos cubos opacos de dimensiones 1× 1× 1 deben ponerse en un cubo

de 3× 3× 3 para asegurar que el cubo completo no es transparente?

(a) 6 (b) 9 (c) 10 (d) 12 (e) 18

Problema 12. Sean a, b, c numeros reales diferentes de cero, que satisfacen:

a+1

b= 2, b+

1

c= 3, c+

1

a= 5.

¿Cual es el valor de abc+1

abc?

(a) 1 (b) 5 (c) 10 (d) 15 (e) 20

Problema 13. Cuando la hormiga va desde la casa siguiendo las flechas→ 3,

↑ 3,→ 3, ↑ 1, llega a la catarina .

Page 27: TZALOA Revista de la Olimpiada Mexicana de Matematicas ... · de un tiempo, Sissa, le presento´ a su rey el juego que le hab´ıa pedido: el ajedrez. Lue-go de entender el juego

Problemas de practica 19

¿A que animal llega si sale de la casa y sigue las flechas: → 2, ↓ 2, → 3, ↑ 3, → 2,

↑ 2?

(a) (b) (c) (d) (e)

Problema 14. ¿Cuantos rectangulos que contengan un numero impar de cuadritos de

1× 1 se pueden encontrar dentro del tablero de 3× 4?

(a) 12 (b) 16 (c) 18 (d) 24 (e) 32

Problema 15. Rocıo tiene los 4 mosaicos en forma de L que se muestran en la figura de

la izquierda y quiere tomar dos de ellos para formar figuras como las que se muestran

a la derecha.

¿Cuantas de esas 4 figuras puede formar?

(a) 4 (b) 3 (c) 2 (d) 1 (e) 0

Problema 16. Ana, Blanca, Ceci y Diana practican cada una deportes distintos: karate,

futbol, volibol y yudo. A Ana no le gustan los deportes de pelota. Blanca practica yudo.

Solo una de las afirmaciones siguientes puede ser verdadera, ¿cual es?

(a) Ana es volibolista (b) Blanca es futbolista (c) Ceci es volibolista

(d) Diana practica karate (e) Ana practica yudo

Problema 17. En cada uno de los puntos • de la figura debes poner un numero, de

manera que la suma de los numeros en los extremos de cada segmento sea la misma.

Page 28: TZALOA Revista de la Olimpiada Mexicana de Matematicas ... · de un tiempo, Sissa, le presento´ a su rey el juego que le hab´ıa pedido: el ajedrez. Lue-go de entender el juego

20 Problemas de practica

4

x

1

b b

b b

b b

b

bb

bb

bb

b

b

b b

b b

b b

b

b b

Dos de los numeros ya se escribieron. ¿Que numero va en el lugar de x?

(a) 1 (b) 3 (c) 4 (d) 5 (e) falta informacion

Problema 18. Azucena tiene 4 tiras de madera de la misma longitud. Pega dos de ellas

con un traslape de 10 cm y ası obtiene una tira de 50 cm de longitud.

10 cm

10 cm

50 cm

Con las otras dos quiere hacer una tira de 56 cm de longitud. ¿Cuanto debe medir el

traslape?

(a) 4 cm (b) 6 cm (c) 8 cm (d) 10 cm (e) 12 cm

Problema 19. La figura se formo con un triangulo equilatero gris de lado 3 cm, dos

triangulos equilateros negros de lado 2 cm y otro triangulo equilatero negro de lado 1cm. La parte comun de dos triangulos se colorea de blanco.

¿Cuanto vale la diferencia del valor del area gris menos el valor del area negra?

(a) −2 (b) −1 (c) 0 (d) 1 (e) 2

Page 29: TZALOA Revista de la Olimpiada Mexicana de Matematicas ... · de un tiempo, Sissa, le presento´ a su rey el juego que le hab´ıa pedido: el ajedrez. Lue-go de entender el juego

Problemas de practica 21

Problema 20. En la figura, se muestra un cuadrilatero ABCD.

A

BC

D

7530

50

Si BC = AD, ¿cuanto mide el angulo ∠ADC?

(a) 30 (b) 50 (c) 55 (d) 65 (e) 70

Problema 21. En un torneo de futbol participan 5 equipos. Cada equipo juega exacta-

mente una vez con cada uno de los otros equipos. En un juego, el ganador obtiene 3puntos y el perdedor 0 puntos, pero si empatan, cada equipo obtiene 1 punto. Si al final

del torneo se suman los puntos de todos los equipos, ¿cuantos valores distintos puede

tener esta suma?

(a) 5 (b) 9 (c) 10 (d) 11 (e) 15

Problema 22. Emilia quiere llenar un tanque para su tortuga y necesita 4 cubetas de

agua para llenarlo. En cada viaje llena la cubeta desde una fuente y camina hacia el

tanque, pero en el camino derrama 13 del contenido de la cubeta. ¿Cuantos viajes tiene

que hacer para llenar el tanque?

(a) 5 (b) 6 (c) 7 (d) 8 (e) 9

Problema 23. En cada partido de futbol de un torneo, al ganador se le otorgaron 3puntos, al perdedor 0 y, si hubo empate, entonces cada equipo gano 1 punto. En 38 par-

tidos un equipo tenıa acumulados 80 puntos. ¿Cual es el maximo numero de partidos

que pudo haber perdido?

(a) 8 (b) 9 (c) 10 (d) 11 (e) 12

Problema 24. ¿Cuantos numeros de tres cifras abc hay que cumplan, 8 > a > b >c > 0?

(a) 8 (b) 35 (c) 42 (d) 70 (e) 210

Problema 25. Un numero entero es bicuadrado si se escribe de manera unica como la

suma de dos cuadrados. Por ejemplo, 25 = 42+32 es bicuadrado, pero 65 = 82+12 =72 + 42 no lo es. ¿Cual de los siguientes numeros es bicuadrado?

(a) 85 (b) 125 (c) 130 (d) 170 (e) 180

Page 30: TZALOA Revista de la Olimpiada Mexicana de Matematicas ... · de un tiempo, Sissa, le presento´ a su rey el juego que le hab´ıa pedido: el ajedrez. Lue-go de entender el juego

Soluciones a los problemas de

practica

Solucion del problema 1. La respuesta es (b). Las demas figuras se pueden construir

como se muestra:

(a) (c) (d) (e)

Solucion del problema 2. La respuesta es (d). Los numeros mas pequenos deben ir a

la izquierda de los grandes; ası, 0, 1 y 2 deben estar a la izquierda de 3 y 3 debe estar a

la izquierda de 8.

Solucion del problema 3. La respuesta es (d). Cada vez que se coloca un cuadrado

chico sobre un lado del grande, se restan 5 cm del perımetro del cuadrado original y

este contribuye con 15 cm al perımetro de la nueva figura. Luego, el perımetro de la

nueva figura es 80− 4(5) + 4(15) = 120 cm.

Solucion del problema 4. La respuesta es (e). En el cubo, cuadrados del mismo color

no pueden compartir un vertice, ası que esto mismo debe ocurrir al desarrollar el cubo.

Entonces, las unicas posibilidades son (d) o (e); sin embargo, al formar el cubo a partir

de (d), las caras mas oscuras quedan compartiendo un vertice. En la opcion (e) quedan

bien.

Otra forma. Las tapas superior e inferior deben ser del mismo color, la (e) es la unica

que lo cumple y tambien sus caras laterales opuestas son del mismo color.

Page 31: TZALOA Revista de la Olimpiada Mexicana de Matematicas ... · de un tiempo, Sissa, le presento´ a su rey el juego que le hab´ıa pedido: el ajedrez. Lue-go de entender el juego

Soluciones a los problemas de practica 23

Solucion del problema 5. La respuesta es (a). El anillo gris debe estar a la izquierda,

ya que de frente esta a la derecha, y debe pasar por arriba del anillo blanco en su parte

izquierda.

Solucion del problema 6. La respuesta es (c). La suma que se ve es 22 por lo que faltan

11 puntos. Junto al 2 va otro 2 y junto al 1 va otro 1, de donde los numeros que faltan

en la posicion marcada con la flecha y el de al lado a la izquierda (que son iguales)

deben sumar 33− 25 = 8. Luego, la respuesta es 82 = 4.

Solucion del problema 7. La respuesta es (d). Los cubitos de la tercera pieza deben

cubrir de la parte de atras, los tres cubitos de abajo y, de la parte de arriba, el cubito

central.

Solucion del problema 8. La respuesta es (b). Denotemos por j, e, p las cantidades de

agua que hay en la jarra, que toma Emilio y que bebe Pablo, respectivamente. Tenemos

por los datos que, 12j = e + p y 4p = (j − e), luego al despejar e se tiene que

12j − p = e = j − 4p, por lo que 1

2j = 3p, de donde p = 16j.

Otra forma. Pablo y Emilio toman media jarra. La otra media jarra equivale a tres

veces lo que bebio Pablo, ya que el tomo la cuarta parte de lo que Emilio dejo. Luego,

si media jarra es tres veces lo que Pablo tomo, entonces Pablo bebio una sexta parte de

la jarra.

Solucion del problema 9. La respuesta es (d). Como cada pieza tiene 5 cuadritos, ne-

cesitamos el menor entero positivo n tal que 5n sea un cuadrado. Tal cuadrado debe

ser divisible entre 5, por lo que n es multiplo de 5. Su otro factor debe ser un cuadrado

diferente de 1 y, como queremos el menor, la respuesta es n = 5 · 22 = 20.

Para formar el cuadrado con las 20 piezas, tomamos primero dos de las piezas y las uni-

mos como en la figura para formar un rectangulo de 5× 2. Con 10 de estos rectangulos

colocando 5 en una fila y 5 mas sobre dicha fila, se obtiene el cuadrado de 10× 10.

Solucion del problema 10. La respuesta es (e). Sentadas las 3 hay 4 espacios entre

ellas donde pueden estar las sillas: A B C pero entre A y B debe haber una silla

y, entre B y C, debe haber otra silla. Luego, las otras 2 sillas deben de acomodarse

en los 4 espacios, lo cual se puede hacer de 6 maneras si solamente acomodamos una

silla en el espacio correspondiente. Pero tambien podemos acomodar las 2 sillas en

uno de los 4 espacios, por lo que hay 10 maneras de acomodar las sillas. Como las

Page 32: TZALOA Revista de la Olimpiada Mexicana de Matematicas ... · de un tiempo, Sissa, le presento´ a su rey el juego que le hab´ıa pedido: el ajedrez. Lue-go de entender el juego

24 Soluciones a los problemas de practica

personas se pueden acomodar de 3! = 6 maneras, resulta que hay 10 · 6 = 60 maneras

de acomodarse.

Solucion del problema 11. La respuesta es (b). Hay 9 filas de cubos en cada una de

las tres direcciones, ası que es necesario tapar 27 direcciones. Como cada cubito tapa 3direcciones, al menos se necesitan 9. Veamos que un acomodo con 9 es posible. En el

siguiente esquema ponemos tres cuadrıculas de 3× 3, cada una de ellas representando

un “piso” del cubo de 3× 3× 3 y sombreamos el lugar donde puede ponerse un cubo,

de manera que se tapen todas las direcciones en el cubo grande. Como el esquema

contiene 9 cuadritos sombreados, entonces 9 es precisamente el mınimo.

Solucion del problema 12. La respuesta es (e). Tenemos que

2 · 3 · 5 =

(

a+1

b

)(

b+1

c

)(

c+1

a

)

= abc+1

abc+

(

a+1

b

)

+

(

b+1

c

)

+

(

c+1

a

)

= abc+1

abc+ 2 + 3 + 5.

Luego, abc+ 1abc = 2 · 3 · 5− (2 + 3 + 5) = 30− 10 = 20.

Solucion del problema 13. La respuesta es (a). En la figura se muestra con lınea gruesa

el camino que sigue la hormiga.

Solucion del problema 14. La respuesta es (d). Los rectangulos con un numero impar

de cuadritos son solamente de los siguientes tamanos: 1 × 1, 1 × 3, 3 × 1 y 3 × 3 de

los que hay 12, 6, 4 y 2, respectivamente. Luego, la respuesta es 24.

Page 33: TZALOA Revista de la Olimpiada Mexicana de Matematicas ... · de un tiempo, Sissa, le presento´ a su rey el juego que le hab´ıa pedido: el ajedrez. Lue-go de entender el juego

Soluciones a los problemas de practica 25

Solucion del problema 15. La respuesta es (a). Todas las formas son posibles.

Solucion del problema 16. La respuesta es (c). Como a Ana no le gustan los deportes

de pelota, sabemos que debe practicar yudo o karate. Pero Blanca practica yudo, ası

que Ana es la que practica karate. Luego, (a), (b) y (e) son falsas. Como las cuatro

practican deportes distintos y Ana practica karate, (d) tambien es falsa. Por lo tanto,

Ceci es volibolista.

Solucion del problema 17. La respuesta es (a). Cuando dos segmentos tienen un punto

en comun, los otros extremos deben tener el mismo valor; ası podemos observar que

todos los puntos marcados con una circunferencia deben llevar el numero 1.

4

x

1

b

b

b

b

b

b

b

b

b

b

b

b

Solucion del problema 18. La respuesta es (a). Las dos tiras juntas miden 50 cm y el

traslape es de 10 cm, ası que cada regla mide 30 cm. Para lograr 56 cm, el traslape debe

de ser de 4 cm pues 56 = 30 + 30− 4.

Solucion del problema 19. La respuesta es (c). El area gris menos el area negra es

igual al area del triangulo gris menos las areas de los triangulos negros, ya que las

partes comunes se suman y restan.

Ahora, notemos que el triangulo gris de lado 3 cm, se puede dividir en 9 triangulos de

lado 1 cm y, el triangulo de lado 2 cm, se puede dividir en 4 triangulos de lado 1 cm.

Como 9 = 4 + 4 + 1, se tiene que la diferencia del area del triangulo gris menos las

areas de los triangulos negros es 0.

Otra manera de terminar. El area de un triangulo equilatero es igual√43 ℓ2, donde ℓ es la

longitud del lado, por lo que la diferencia de las areas es,√43 ·32−2·

√43 ·22−

√43 ·12 = 0.

Solucion del problema 20. La respuesta es (d). Tenemos que ∠BAC = 180− 75−30 = 75, ası que AC = BC = AD; es decir, el triangulo ACD es isosceles y

entonces ∠ACD = ∠ADC. Por lo anterior, ∠ADC = (180 − 50)/2 = 65.

Solucion del problema 21. La respuesta es (d). El numero de partidos del torneo es

10. En un partido se dan 3 puntos entre los dos equipos si hay un ganador y se dan 2

Page 34: TZALOA Revista de la Olimpiada Mexicana de Matematicas ... · de un tiempo, Sissa, le presento´ a su rey el juego que le hab´ıa pedido: el ajedrez. Lue-go de entender el juego

26 Soluciones a los problemas de practica

puntos si hay empate. El maximo de puntos es 30 y sucede cuando no hay empates. La

menor suma es 20 y ocurre cuando hay empate en cada uno de los 10 partidos. Luego,

hay 11 posibles sumas que son 30−x ·1, donde x es el numero de empates en el torneo

que puede ser cualquiera de los valores 0, 1, 2, . . . , 10.

Solucion del problema 22. La respuesta es (b). En cada viaje, Emilia llena 23 de la

cubeta. Como 6× 23 = 4, Emilia necesita hacer 6 viajes para completar 4 cubetas.

Otra forma: Cada 3 viajes completa 2 cubetas, luego necesita hacer 6 viajes.

Solucion del problema 23. La respuesta es (c). Llamemos g al numero de partidos

ganados, e al de empatados y p al de perdidos. Tenemos que 80 = 3g + e y 38 =g+e+p. Multiplicando la segunda ecuacion por 3, obtenemos que 114 = 3g+3e+3p.

Si a esta ecuacion le restamos la primera ecuacion, obtenemos que 34 = 2e+3p. Como

ambos e y p son no negativos, tenemos que p ≤ 11 pero si p = 11, entonces e no es

entero; para p = 10 tenemos e = 2 y, sustituyendo en la primera ecuacion, g = 26.

Esta es la solucion de ambas ecuaciones que tiene la maxima p.

Solucion del problema 24. La respuesta es (b). Numeros de tres cifras distintas y con

cifras menores a 8, hay 7 · 6 · 5 = 210. Pero de estos numeros hay 6 que se forman con

las mismas cifras y solamente hay uno que cumple que a > b > c. Luego, la respuesta

es 2106 = 35.

Solucion del problema 25. La respuesta es (e). Tenemos que 180 = 122 + 62. Los

cuadrados menores a 180 son 12, 22, . . . , 122, 132. Para a = 1, 2, . . . , 13, se tiene que

180− a2 es cuadrado solamente cuando a = 6 o a = 12.

Los otros numeros no son bicuadrados, ya que 85 = 62 + 72 = 22 + 92, 125 =22 + 112 = 52 + 102, 130 = 32 + 112 = 72 + 92 y 170 = 12 + 132 = 72 + 112.

Page 35: TZALOA Revista de la Olimpiada Mexicana de Matematicas ... · de un tiempo, Sissa, le presento´ a su rey el juego que le hab´ıa pedido: el ajedrez. Lue-go de entender el juego

Problemas de Entrenamiento

Problemas de Entrenamiento.

Ano 2019 No. 1.

Presentamos ahora los 10 problemas de entrenamiento elegidos para este numero de

tu revista. Te recordamos que las soluciones de los problemas en esta seccion no las

publicamos en este momento, por lo que te invitamos a que los resuelvas y nos envıes

tus soluciones. Las soluciones de los problemas de esta seccion se escogeran de entre

las participaciones recibidas por parte de la comunidad olımpica de todo el paıs.

Con el fin de dar tiempo a nuestros lectores para la redaccion y envıo de sus tra-

bajos, las soluciones de los problemas presentados en cada numero de la revista, se

publican 3 numeros despues. Para ello, ponemos a tu disposicion nuestra direccion:

[email protected] y ten la seguridad de que tan pronto recibamos tu con-

tribucion, inmediatamente nos pondremos en contacto contigo para comentar y en su

caso, publicar tu trabajo. ¡Te invitamos a intentarlo!

Problema 1. Esteban vende galletas en cajas pequenas de 5 galletas y en cajas grandes

de 12 galletas. Tiene muchas cajas de cada tipo, pero no hay cajas de distintos tamanos

y no vende galletas sueltas. Si, por ejemplo, un cliente quiere 39 galletas, Esteban

puede despachar el pedido exactamente con tres cajas pequenas y dos grandes, ya que

3×5+2×12 = 39. Pero hay pedidos que no se pueden despachar de manera exacta, por

ejemplo, cuando un cliente quiere 7, 16 o 23 galletas. ¿Cual es el pedido mas grande

que no se puede despachar de manera exacta?

Problema 2. Determina todos los numeros primos p, q y r, distintos entre sı, tales que

p3 − q2 = r y p+q+rq = r.

Problema 3. Sean a y b numeros reales distintos entre sı y distintos de cero tales quea−2010

b + b+2010a = 2. Determina el valor de a− b.

Page 36: TZALOA Revista de la Olimpiada Mexicana de Matematicas ... · de un tiempo, Sissa, le presento´ a su rey el juego que le hab´ıa pedido: el ajedrez. Lue-go de entender el juego

28 Problemas de Entrenamiento

Problema 4. En la figura, ABCDEF es un hexagono regular de area 2018 y G es el

punto medio del lado AB. Calcula el valor del area sombreada.

E D

C

BA

F

G

HI

Problema 5. Un torneo de 215 jugadores es de eliminacion directa, esto es, en cuanto

un jugador pierde un juego, sale del torneo. Ası, en la primera ronda hay 107 juegos y

un jugador pasa directamente. En la segunda ronda hay 108 jugadores y por tanto hay

54 juegos sin que nadie pase directamente, y ası sucesivamente. ¿Cuantos juegos hay?

Problema 6. Sean x, y, z numeros reales positivos. Demuestra que

x2 + xy2 + xyz2 ≥ 4xyz − 4.

Problema 7. Sea ABCD un cuadrilatero convexo y sean K , L, M , N los puntos

medios de los lados AB, BC, CD, DA, respectivamente. Si BD parte a KM por la

mitad en Q, QA = QB = QC = QD y LKLM = CD

CB , demuestra que ABCD es un

cuadrado.

Problema 8. Determina si existen 2019 enteros positivos a1 < a2 < · · · < a2019 tales

que mcd(ai, aj) = aj − ai para cualesquiera i, j con 1 ≤ i < j ≤ 2019.

Problema 9. Sean ABC un triangulo y M el punto medio de AC. La circunferencia

tangente a BC porB que pasa porM interseca a la recta AB de nuevo enP . Demuestra

que AB · BP = 2BM2.

Problema 10. Los enteros positivos del 1 al 49 se colocan al azar en las casillas de un

tablero de ajedrez de 7× 7. Demuestra que hay un subtablero de 2× 2 tal que la suma

de los cuatro numeros en sus casillas es por lo menos 81.

Soluciones a los Problemas de Entrenamiento.

Ano 2018 No. 2.

A continuacion presentamos las soluciones de los problemas de entrenamiento pro-

puestos en Tzaloa No. 2, ano 2018. Aprovechamos para invitar a todos los lectores a

Page 37: TZALOA Revista de la Olimpiada Mexicana de Matematicas ... · de un tiempo, Sissa, le presento´ a su rey el juego que le hab´ıa pedido: el ajedrez. Lue-go de entender el juego

Problemas de Entrenamiento 29

participar enviandonos sus soluciones para que puedan salir publicadas en los numeros

posteriores de la revista. Recuerda que en el siguiente numero de la revista apareceran

las soluciones de los problemas de entrenamiento propuestos en Tzaloa No. 3, ano

2018, por lo que aun tienes tiempo de enviarnos tus soluciones.

Problema 1. Sea O el centro del plano cartesiano. Se toma un punto A en el primer

cuadrante con coordenadas (a, b) y se traza el segmento OA. Se gira OA en direc-

cion contraria a las manecillas del reloj hasta un punto B con coordenadas (b, a). Si

∠AOB = 30, encuentra el valor de ab .

Solucion. Supongamos, sin perdida de generalidad, que a > b. Marcamos a y b en

los ejes x, y y llamamos P y Q a los puntos (a, 0) y (0, a), respectivamente. Por el

criterio de congruencia LLL, los triangulosAOP y BOQ son congruentes, pues ambos

tienen lados que miden OA, a y b. Luego, los angulos ∠AOP,∠BOQ son iguales y

suman 60. Como ∠APO = ∠BQO = 90, los triangulos AOP y BOQ son medios

equilateros y AO = 2b. Por el teorema de Pitagoras, a =√4b2 − b2 =

√3b, de donde

obtenemos que ab =√3.

Problema 2. Encuentra todas las ternas (x, y, z) de enteros no negativos tales que

x2 + yz = 1, y2 + xz = 2 y z2 + xy = 4.

Solucion. Duplicamos cada una de las tres ecuaciones y las sumamos para obtener

2x2 + 2yz + 2y2 + 2xz + 2z2 + 2xy = 14. Si separamos de manera conveniente,

obtenemos que x2 + 2xy + y2 + x2 + 2xz + z2 + y2 + 2yz + z2 = 14, que puede

escribirse como (x + y)2 + (x + z)2 + (y + z)2 = 14. Como los cuadrados son no

negativos y 42 = 16, nuestras opciones son 1, 4 y 9. Pero 4+4+4 < 14, de modo que

la unica solucion es 1 + 4 + 9 = 14. Suponemos que a+ b = 1, b+ c = 2, c+ a = 3.

Resolvemos el sistema de ecuaciones y obtenemos que b = 0, a = 1, c = 2. Para que

esto tenga sentido en el sistema original, tenemos x = 1, y = 0, z = 2, que es la unica

solucion posible.

Problema 3. Andrea tiene 7 cuyos de los cuales 3 son hembras y 4 son machos. Ella

quiere repartirlos en 4 jaulas distintas (ninguna jaula quedara vacıa). ¿De cuantas ma-

neras puede hacer esto si quiere que, donde haya al menos una hembra, no haya ningun

macho? Nota: Los cuyos son diferentes entre sı.

Solucion. Separamos en 3 casos segun la cantidad de jaulas que ocupan las hembras.

a) Caso 1. Si las hembras ocupan solo una jaula, quedan 3 jaulas para los machos. Las

hembras tienen una unica manera de acomodarse. Hay 4 maneras de elegir la jaula

de las hembras, hay 3 maneras de elegir cual jaula tendra 2 machos, hay 6 maneras

de elegir cual pareja de machos la compartira, y hay 2 maneras en que los otros dos

machos ocupen las jaulas restantes. Son 4× 36 = 144 maneras en este caso.

b) Caso 2. Si las hembras ocupan 2 jaulas, quedan 2 jaulas para los machos. Hay 6maneras de elegir cuales dos jaulas ocupan las hembras, 2 maneras de elegir cual

jaula tendra unicamente una hembra y 3 maneras de elegir cual hembra la ocupara.

Page 38: TZALOA Revista de la Olimpiada Mexicana de Matematicas ... · de un tiempo, Sissa, le presento´ a su rey el juego que le hab´ıa pedido: el ajedrez. Lue-go de entender el juego

30 Problemas de Entrenamiento

Para dividir a los machos hay 14 maneras: 6 maneras si los separamos dos y dos,

y 8 maneras si los separamos uno y tres (2 maneras de elegir la jaula con uno y 4maneras de elegir al macho que ocupa). Luego, en este caso son 36× 14 = 504.

c) Caso 3. Si las hembras ocupan 3 jaulas, queda una jaula para los machos. Los ma-

chos tienen una unica manera de acomodarse. Hay 4 maneras de elegir la jaula de

los machos y hay 6 maneras en que pueden acomodarse las hembras. Son 24 mane-

ras en este caso.

En total son 144 + 504 + 24 = 672 maneras de acomodar a los cuyos.

Problema 4. Si escribimos la secuencia AAABABBB a lo largo del perımetro de un

cırculo, cada palabra de longitud 3 que consiste en las letras A y B (es decir, AAA,

AAB, ABA, BAB, ABB, BBB, BBA, BAA) aparece exactamente una vez en el

perımetro. Muestra que es posible escribir una secuencia de letras de un alfabeto de kelementos a lo largo del perımetro de un cırculo, de tal manera que cada palabra de

longitud ℓ (es decir, una ℓ-tupla de letras ordenada) aparezca exactamente una vez en

el perımetro.

Solucion. Denotemos al alfabeto por P y consideremos la grafica dirigida G = (V,E),donde

V = [a1, . . . , aℓ−1] : ai ∈ P,E = [[a1, . . . , aℓ−1][b1, . . . , bℓ−1]] : a2 = b1, a3 = b2, . . . , aℓ−1 = bℓ−2.

Si tomamos dos vertices [a1, . . . , al−1] y [b1, . . . , bl−1] veamos que debe haber al me-nos un camino orientado entre ellos:

[a1, . . . , aℓ−1]←→ [a2, . . . , aℓ−1, b1]←→ [a3, . . . , aℓ−1, b1, b2]←→ · · · ←→ [b1, . . . , bℓ−1]

(algunos vertices y aristas pueden repetirse en la secuencia). Lo anterior implica que

la grafica es fuertemente conexa. Por otro lado, cada vertice [a1, . . . , aℓ−1] tiene exac-

tamente k arcos salientes y k arcos entrantes: los arcos salientes estan dirigidos a los

vertices [a2, . . . , aℓ−1, o], donde o pasa por todo el alfabeto P y los entrantes vienen de

los vertices [i, a1, . . . , aℓ−2], donde i tambien pasa por todo el alfabeto. Eso significa

que los grados internos y externos de cada vertice son iguales, entonces la grafica es una

grafica de Euler (dirigida). Como consecuencia, existe un ciclo Euleriano en la grafica,

es decir, un ciclo que contiene todas las aristas y cada arista aparece exactamente una

vez. Ası podemos formar la secuencia cıclica buscada de la siguiente manera: Comen-

cemos con un vertice arbitrario y escribamos su secuencia a1, . . . , aℓ−1. Sigamos el

ciclo de Euler y agreguemos la ultima letra de cada vertice a la secuencia hasta que

lleguemos al primer vertice de nuevo. Ahora borramos las ultimas ℓ − 1 letras (que

son necesariamente las mismas que las iniciales). Dado que hay una biyeccion entre el

conjunto de todas las palabras de ℓ letras y el conjunto de aristas de V :

[a1, . . . , aℓ]←→ [[a1, . . . , aℓ−1], [a2, . . . , aℓ]]

se sigue que la secuencia tiene las propiedades requeridas.

Page 39: TZALOA Revista de la Olimpiada Mexicana de Matematicas ... · de un tiempo, Sissa, le presento´ a su rey el juego que le hab´ıa pedido: el ajedrez. Lue-go de entender el juego

Problemas de Entrenamiento 31

Problema 5. Sean a y d dos enteros positivos. Demuestra que existe una constan-

te K tal que cada conjunto de K elementos consecutivos de la progresion aritmetica

an∞n=1 = a+ nd∞n=1 contiene al menos un numero que no es primo.

Solucion. Sea p un numero primo tal que p no divide a d. Supongamos que para algun

entero positivo n se tiene que los elementos an, an+1, . . . , an+p son primos. Clara-

mente se tiene que an+i = an + id. Entonces an > p porque an+an= an + and no

es un numero primo. Luego, en el sistema de residuos an+1, . . . , an+p (mod p), como

cada primo es mayor que p, no esta el numero 0. Ası, existen dos numeros r1 < r2tales que ar1 ≡ ar2 (mod p). Esto quiere decir que an + r1d ≡ an + r2d (mod p) y ası

r1d ≡ r2d (mod p), pero como mcd(p, d) = 1, entonces r1 ≡ r2 (mod p), lo cual es

un absurdo.

Problema 6. Sea δ(n) el maximo divisor impar de n, con n un entero positivo. De-

muestra que para todo entero positivo m,

S(m)− 2m

3

< 1,

donde S(m) =∑m

n=1δ(n)n .

Solucion. Notemos que δ(2k+1) = 2k+1, δ(2k) = δ(k) y S(2k+1) = S(2k) + 1.

Luego, tenemos que

S(2k) =

2k∑

n=1

δ(n)

n=

k∑

n=1

δ(2n)

2n+

k∑

n=1

δ(2n− 1)

2n− 1

=1

2

k∑

n=1

δ(n)

n+

k∑

n=1

2n− 1

2n− 1=

1

2S(k) + k.

Si F (m) = S(m) − 2m3 , entonces F (2k) = S(2k) − 4k

3 = 12S(k) + k − 4k

3 =12S(k)− k

3 = 12F (k) y F (2k+ 1) = S(2k+ 1)− 2(2k+1)

3 = S(2k) + 1− 2(2k+1)3 =

12S(k) + k+1− 4

3k− 23 = 1

2S(k)− k3 + 1

3 = F (2k) + 13 . Finalmente, un argumento

inductivo muestra que 0 < F (m) ≤ 23 para todo entero positivo m, de donde se sigue

el resultado.

Problema 7. Determina todas las parejas de enteros positivos (a, n) con a ≥ n ≥ 2,

tales que (a+ 1)n + a− 1 sea una potencia de 2.

Solucion. Aplicando el teorema del binomio, tenemos que

(a+ 1)n + a− 1 = an + · · ·+ n(n− 1)

2a2 + na+ 1+ a− 1

= an + · · ·+ n(n− 1)

2a2 + (n+ 1)a. (1)

Page 40: TZALOA Revista de la Olimpiada Mexicana de Matematicas ... · de un tiempo, Sissa, le presento´ a su rey el juego que le hab´ıa pedido: el ajedrez. Lue-go de entender el juego

32 Problemas de Entrenamiento

Luego, como todos los terminos son divisibles por a y (a+1)n+a−1 es una potenciade 2, se sigue que a tambien es una potencia de 2. Sean a = 2b y (a+1)n+a+1 = 2c.Como a ≥ 2, tenemos que b ≥ 1. Mas aun, ya que n ≥ 2, resulta que 2c > a2 = 22b

y, por lo tanto, c > 2b.Note que todos los terminos de (1) excepto el ultimo, son divisibles por a2 = 22b.Como c > 2b, tenemos que 2c es divisible por 22b y (n + 1)a tambien lo es. Comoa = 2b, se sigue que 2b divide a n+ 1, esto es, n+ 1 = 2bm = am para algun enteropositivo m. Dado que a ≥ n ≥ 2, el unico valor posible de m es m = 1 y, por lo tanto,n = a− 1 = 2b − 1. Si b = 1, entonces n = 1, lo cual no puede ser. Luego, b > 1 locual implica que a ≥ 4 y n = a− 1 ≥ 3. De aquı que 2c = (a+ 1)n + a− 1 > an ≥a3 = 23b, lo cual implica que c > 3b.Entonces,

(a+ 1)n + a− 1 = an + · · ·+

n(n− 1)(n− 2)

6a3 +

n(n− 1)

2a2 + (n+ 1)a

= 2nb + · · ·+(2b − 1)(2b − 2)(2b − 3)

623b +

(2b − 1)(2b − 2)

222b + 22b.

Todos los terminos, excepto los ultimos dos son divisibles por 23b. Mas aun, 2c es

divisible por 23b y, por lo tanto,(2b−1)(2b−2)

2 22b + 22b = (2b − 1)(2b−1 − 1)22b + 22b

tambien es divisible por 23b.Se sigue que (2b − 1)(2b−1 − 1) + 1 = 22b−1 − 2b − 2b−1 + 2 es divisible por 2b,pero esto solo es posible cuando b = 2: si b > 2, todos los terminos excepto el ultimo,

son divisibles por 4 y, en consecuencia, la suma no es divisible entre 4 (y tampoco es

divisible por 2b).Si b = 2, entonces a = 4 y n = 3. En este caso es facil verificar que (a+1)n+a−1 =128 = 27. Por lo tanto, la unica solucion es a = 4 y n = 3.

Problema 8. Se escoge un punto K en la diagonal de un cuadrilatero convexo ABCDde forma que KD = DC, ∠BAC = 1

2∠KDC y ∠DAC = 12∠KBC. Prueba que

∠KDA = ∠BCA o ∠KDA = ∠KBA.

Solucion. Sea X el punto de interseccion entre la bisectriz del angulo ∠CDK y la

recta AB. Como KD = DC, entonces la bisectriz del angulo ∠CDK tambien es

la mediatriz del triangulo KDC; por tanto, XD es la bisectriz y la mediatriz del

triangulo KCX . Ademas, ∠BAC = ∠XAC = 12∠KDC = ∠XDC; en con-

secuencia, XCDA es un cuadrilatero cıclico. Ası, ∠DXC = ∠DAC. Por tanto,

∠KXC = 2∠DAC = ∠KBC. Sea Y el punto de interseccion (distinto de X)

entre el circuncırculo del triangulo XCK y la recta AB. Por ser XCKY cıclico,

∠KY C = ∠KXC = ∠KBC. Por tanto, X e Y son los unicos dos puntos en ABque satisfacen que ∠KXC = ∠KY C = 2∠DAC. Pero, ∠DAC = 1

2∠KBC, ası

que B debe ser X o Y . En consecuencia,

∠KDA = 180 − ∠DKA− ∠KDA = ∠CKD − ∠KAD

= 90 − 1

2∠KDC − ∠KAD = 90 − ∠XAC − ∠CAD

= 90 − ∠XAC − 1

2∠KXC.

Page 41: TZALOA Revista de la Olimpiada Mexicana de Matematicas ... · de un tiempo, Sissa, le presento´ a su rey el juego que le hab´ıa pedido: el ajedrez. Lue-go de entender el juego

Problemas de Entrenamiento 33

Luego, ∠KXA = ∠CKX − ∠XAC = 90 − 1

2∠KXC − ∠XAC y, por lo tanto,

∠KXA = ∠Y CA. Entonces, ∠KDA = ∠KXA = ∠Y CA. Para concluir basta con

sustituir en los casos cuando B = X o cuando B = Y .

Problema 9. Encuentra el menor entero positivo n con la siguiente propiedad: para

todo conjunto finito X de puntos en el plano, si por cada m ≤ n puntos del conjunto

existen dos lıneas que contienen a todos los m puntos, entonces existen dos lıneas que

contienen a todos los puntos de X .

Solucion. La siguiente configuracion muestra que n ≥ 6. En efecto, cada conjunto de

5 o menos puntos esta contenido en dos lıneas, pero el conjunto completo no lo esta.

•• •• • •

Ahora, demostraremos que con n = 6 sı se puede. Sea X un conjunto de puntos en

el plano, si | X |< 6, por hipotesis todos los puntos estan en dos rectas. Si | X |≥ 6,

entonces es posible tomar seis puntos y, por hipotesis, estos estaran contenidos en dos

rectas. En particular, hay tres puntos en una misma recta Γ. Ahora, supongamos que

existen dos puntos x, y ∈ X que no estan contenidos en Γ, de otra forma ya se habrıa

terminado. Sea Λ la recta que pasa por x, y. Supongamos que existe z ∈ X tal que no

este en Γ o en Λ, de otra forma ya se habrıa terminado. Por la hipotesis del problema,

los tres puntos de Γ, los dos de Λ y z estan en dos rectas. Sin embargo, x y y no estan

en Γ, entonces las dos rectas en las que estan los seis puntos anteriores son Γ y Λ; lo

cual es una contradiccion, pues z no estaba en esas rectas. Por lo tanto, todos los puntos

estan en Γ o en Λ.

Problema 10. Determina todas las funciones f : R −→ R tales que para cualesquiera

numeros reales x, y ∈ R se satisface f(xf(y)− yf(x)) = f(xy)− xy.

Solucion. Al sustituir y = 0, se obtiene que f(xf(0)) = 0. Si f(0) 6= 0, la expresion

xf(0) toma todos los valores reales posibles; en consecuencia, f es una funcion cons-

tante, lo cual es imposible. Por tanto, f(0) = 0.

Si se sustituye y = x, se obtiene que f(0) = f(x2) − x2, es decir, f(x2) = x2. Por

tanto, f(x) = x para todo real x ≥ 0. Ahora, sean x, y < 0. Ası, f(xy) = xy, por

lo que f(xf(y) − yf(x)) = 0, lo cual solo puede ocurrir si xf(y) − yf(x) ≤ 0. De

manera analoga se tiene que yf(x)−xf(y) ≤ 0, es decir, yf(x)−xf(y) = 0. En con-

secuencia,f(x)x = f(y)

y . Se sigue que f(x) = cx para todo x < 0 con c una constante.

Ahora, para x < 0 < y se tiene que f(xf(y) − yf(x)) = f(xy − cxy) = f((1 −c)xy) = f(xy) − xy = (c− 1)xy. Ası, f(z) = −z para z = (1 − c)xy. Si c = 1, se

obtiene la solucion trivial f(x) = x para todo x ∈ R. Si c 6= 1, basta sustituir x = −1y y = 1 en z = (1 − c)xy = c − 1 para obtener que f(c − 1) = −(c − 1). Ası,

c − 1 < 0, entonces f(c − 1) = c(c − 1). Por tanto, −(c − 1) = c(c − 1), es decir,

c = −1. En este caso se tiene que f(x) = |x|. Una verificacion directa confirma que

esta ultima solucion funciona; en efecto, si x > 0 > y (los demas casos son analogos),

se tiene que−2xy = f(−2xy) = f(xf(y)− yf(x)) = f(xy)− xy = −2xy.

Page 42: TZALOA Revista de la Olimpiada Mexicana de Matematicas ... · de un tiempo, Sissa, le presento´ a su rey el juego que le hab´ıa pedido: el ajedrez. Lue-go de entender el juego

2a Olimpiada Mexicana de

Matematicas para Educacion

Basica, Concurso Nacional

Del 9 al 12 de junio de 2018 se llevo a cabo, en Merida, Yucatan, la 2a Olimpiada

Mexicana de Matematicas para Educacion Basica (OMMEB) en los niveles de primaria

y secundaria, con la participacion de 261 estudiantes representando a 29 entidades

federativas. La OMMEB esta compuesta de tres niveles:

a) Nivel I: Estudiantes de cuarto y quinto ano de primaria o una institucion equivalente.

b) Nivel II: Estudiantes de sexto ano de primaria y de primer ano de secundaria o una

institucion equivalente.

c) Nivel III: Estudiantes de segundo ano de secundaria o una institucion equivalente.

Hay dos tipos de examenes: individual y por equipos. El nivel I de la prueba individual

consto de 15 problemas para resolver en 90 minutos. Los niveles II y III de la prueba

individual constaron de 15 problemas para resolver en 120 minutos. Los problemas se

dividen en dos partes. La parte A consiste de 12 problemas. La parte B consiste de 3

problemas de redaccion libre. En los tres niveles, la prueba por equipos consistio de 8

problemas, a resolver en 70 minutos.

Los ganadores de medalla de oro y medalla de plata en cada categorıa, integran la

preseleccion nacional, a partir de la cual se formaran los equipos que representaran

a Mexico en la Competencia Internacional de Matematicas (IMC), a celebrarse en el

verano de 2019.

A continuacion, listamos los nombres de los alumnos que obtuvieron medalla de oro

en cada nivel de la competencia.

Page 43: TZALOA Revista de la Olimpiada Mexicana de Matematicas ... · de un tiempo, Sissa, le presento´ a su rey el juego que le hab´ıa pedido: el ajedrez. Lue-go de entender el juego

2a OMMEB, Concurso Nacional 2018 35

Nombre Estado Nivel

Alejandro M. Roque Laparra Chiapas I

Mateo I. Latapı Acosta Ciudad de Mexico I

Sebastian Montemayor T. Nuevo Leon I

Zizou Rueda Galindo Oaxaca I

Dahiana Y. Arvizu Islas Tamaulipas I

Diego Caballero Ricaurte Ciudad de Mexico II

Fidan Garaev Garayeva Michoacan II

Luis E. Martınez Aguirre Nuevo Leon II

Alier Sanchez y Sanchez Quintana Roo II

Victor M. Bernal Ramırez Sinaloa II

Jacobo De Juan Millon Yucatan II

Leonardo M. Cervantes M. Ciudad de Mexico III

Ana Illanes M. de la Vega Ciudad de Mexico III

Diego A. Villarreal Grimaldo Nuevo Leon III

Samantha Ruelas Valtierra Queretaro III

Daniel A. Ochoa Quintero Tamaulipas III

A continuacion, listamos los nombres de los alumnos que obtuvieron medalla de plata

en cada nivel de la competencia.

Nombre Estado Nivel

Daniel Elıas Navarrete Flores Chihuahua I

Javier Caram Quiros Ciudad de Mexico I

Raul E. Flores Renterıa Coahuila I

Rodrigo Aviles Cabrera Guanajuato I

Said Huizar Dorantes Guanajuato I

Cristofer Sosa Gutierrez Hidalgo I

Jaziz Cortes Camiro Michoacan I

Yeshua A. Wong Vargas Morelos I

Santiago Polendo Perini Nuevo Leon I

Carlota Ordonez Bravo Quintana Roo I

Eduardo A. Esparza C. San Luis Potosı I

Nicolas Santana Bon Sinaloa I

Luis Angel G. Jimenez Iturbide Tabasco I

Leticia Perez Rodrıguez Tabasco I

Enrique Jackson Ajuria Yucatan I

Juan P. Espinosa Martınez Zacatecas I

Carlos F. Martınez Quintero Ciudad de Mexico II

Rosa V. Cantu Rodrıguez Ciudad de Mexico II

Jose R. Gutierrez Suarez Colima II

Juan B. Olivares Rodrıguez Guanajuato II

Cynthia N. Lopez Estrada Guanajuato II

Omar F. Astudillo Marban Guerrero II

Page 44: TZALOA Revista de la Olimpiada Mexicana de Matematicas ... · de un tiempo, Sissa, le presento´ a su rey el juego que le hab´ıa pedido: el ajedrez. Lue-go de entender el juego

36 2a OMMEB, Concurso Nacional 2018

Nombre Estado Nivel

Diego Ocaranza Nunez Jalisco II

Pedro E. Mata Castanuela Nuevo Leon II

Fernando Alvarez Ruız Nuevo Leon II

David Garcıa Maldonado Oaxaca II

Valentina Acosta Bueno San Luis Potosı II

Tiago I. Vargas Rivera Yucatan II

Marıa F. Lopez Tuyub Yucatan II

Daniel Canas Urbina Chiapas III

Hector Lomelı Garcıa Ciudad de Mexico III

Adrian A. Garcıa Lopez Jalisco III

Gerardo Padilla Gonzalez Jalisco III

Shubham S. Kumar Agarwal Morelos III

Uriel J. Hernandez Guzman Nuevo Leon III

Abel Arizpe Kisfalusi Nuevo Leon III

Monica I. Casillas Rodrıguez Queretaro III

Rodrigo Gaeta Lopez San Luis Potosı III

Karla R. Munguıa Romero Sinaloa III

Guillermo C. Gruintal Polanco Yucatan III

Marco A. Olivares Amaro Zacatecas III

A continuacion presentamos los problemas y soluciones del concurso nacional de la 2a

OMMEB.

Pruebas Individuales

Nivel I.

1) En cuatro dıas, seis maquinas impresoras han impreso 100 libros. ¿Cuantos dıas

tardaran en imprimir 50 libros si solo funcionan cuatro maquinas impresoras?

2) La siguiente serpiente tiene 2018 cuadritos que se han pintado de tres colores si-

guiendo el patron: blanco, gris, negro, blanco, gris, negro, etc. ¿Cuantos cuadritos

grises hay?

b b b

3) A un club de matematicas asisten 37 estudiantes. Si las ninas se pueden dividir en

equipos de 8 sin que sobre ninguna y los ninos se pueden dividir en equipos de 7ninos sin que sobre ninguno, ¿cuantas ninas hay en el club?

4) Decimos que un numero natural es yucateco si tiene 9 dıgitos, todos son diferentes

y ninguno de ellos es cero. ¿Cual es la menor diferencia positiva posible entre dos

numeros yucatecos?

Page 45: TZALOA Revista de la Olimpiada Mexicana de Matematicas ... · de un tiempo, Sissa, le presento´ a su rey el juego que le hab´ıa pedido: el ajedrez. Lue-go de entender el juego

2a OMMEB, Concurso Nacional 2018 37

5) Mary tiene sus ahorros en una alcancıa y decide gastarlos de la siguiente manera: El

primer dıa gasta 20 pesos, el segundo gasta 21 pesos, el tercero 22 pesos, el cuarto

23 pesos y ası sucesivamente, de tal modo que cada dıa gasta un peso mas que el dıa

anterior. El dıa 18 al ir a sacar sus monedas, se da cuenta que tiene en su alcancıa

exactamente un peso mas que lo que gasto el dıa anterior, ¿cuanto tenıa ahorrado

Mary?

6) Un entero positivo n se dice que es maya si en la siguiente lista de numeros enteros

consecutivos 101, 102, 103, . . . , 200, hay exactamente un multiplo de n. Encuentra

el numero maya mas pequeno.

7) La siguiente figura se construyo con palillos de madera de la misma longitud. Si el

perımetro del triangulo mayor es 96 cm, ¿cual es la suma de las longitudes, en cm,

de todos los palillos usados?

8) La fraccion 28 es equivalente a 1

4 , y cuando agregas 1 tanto al numerador como al de-

nominador de 28 obtienes 3

9 , que es equivalente a 13 . Encuentra una fraccion que sea

equivalente a 18 , de manera que cuando agregues 1 al numerador y al denominador

de tu fraccion, obtengas una fraccion equivalente a 17 .

9) Considera un trapecio ABCD, con los lados BC y DA paralelos y con CD =DA = AB = 1

2BC. Encuentra la medida en grados del angulo ∠CAB.

A

B C

D

10) Si un triangulo equilatero y un hexagono regular tienen el mismo perımetro y el

area del hexagono es de 120 cm2, ¿cual es el area, en cm2, del triangulo?

11) En una pared esta escrita la palabra YUCATAN con letras de metal. Al menos una

de las letras se cayo, pero no se cayeron todas. ¿Cuantas palabras distintas pueden

haber quedado escritas en la pared, sin considerar los espacios vacıos? Por ejemplo,

si se cayeron la C y la T, queda YUAAN.

12) Un cırculo se colorea de gris y blanco, y sobre la circunferencia estan marcados 6puntos, como se indica en la figura.

Page 46: TZALOA Revista de la Olimpiada Mexicana de Matematicas ... · de un tiempo, Sissa, le presento´ a su rey el juego que le hab´ıa pedido: el ajedrez. Lue-go de entender el juego

38 2a OMMEB, Concurso Nacional 2018

bb

b

b b

b

Decimos que un cuadrilatero es bicolor si su interior tiene una parte blanca y una

parte gris. ¿Cuantos cuadrilateros bicolor tienen sus cuatro vertices en los puntos

marcados?

13) En un baile de la escuela, cada alumno bailo con 3 alumnas y cada alumna bailo

con 6 alumnos. Si al baile asistieron 90 personas entre alumnas y alumnos, ¿cuantos

alumnos fueron al baile?

14) Un rectangulo se divide en tres rectangulos mas pequenos como se muestra en la

figura. Cada uno de los rectangulos mas pequenos cumple que sus lados estan en la

misma proporcion que los lados del rectangulo grande. En cada uno de los cuatro

rectangulos, ¿cual es la razon de la longitud del lado mas grande entre la longitud

del lado mas pequeno?

15) Hugo escribe en su libreta exactamente una vez cada uno de los numeros de la forma

1±2±3±4±· · ·±10. Por ejemplo, uno de ellos es 1−2+3+4−5−6−7−8+9−10.

Encuentra la suma de todos estos numeros.

Nivel II. Parte A.

1) ¿Cuantos numeros primos dividen a 732 − 312 − 91?

2) La siguiente figura se formo con dos cuadrados de lado 1 cm, el ABCD y el

AB′C′D′, de manera que AB′ esta sobre la diagonal AC. Sea E el punto de inter-

seccion de B′C′ con CD.

A B

C

D′

C′

D

B′

E

Encuentra el area, en cm2, del cuadrilatero AB′ED.

Page 47: TZALOA Revista de la Olimpiada Mexicana de Matematicas ... · de un tiempo, Sissa, le presento´ a su rey el juego que le hab´ıa pedido: el ajedrez. Lue-go de entender el juego

2a OMMEB, Concurso Nacional 2018 39

3) Coincide con el Problema 13 del Nivel I.

4) Coincide con el Problema 14 del Nivel I.

5) Isaac y Alfredo juegan a lanzar dados de la siguiente manera. Isaac lanza un dado

y apunta el numero que salio en su libreta, luego vuelve a lanzar el dado y apunta

el numero que le salio a la derecha del numero que ya habıa escrito, formando ası

un numero de 2 dıgitos. Luego, Alfredo hace lo mismo que hizo Isaac. ¿Cual es la

probabilidad de que el numero de Alfredo sea mayor que el numero de Isaac?

6) Sean ABC un triangulo rectangulo con ∠ABC = 90, D un punto que cumple que

BDC y ABC son triangulos semejantes, ademas A y D estan en lados opuestos de

BC. El punto E sobre CD cumple que los angulos ∠CAE y ∠EAB son iguales.

Si AE es paralelo a BD, ¿cuanto mide (en grados) el angulo ∠CAB?

A

B C

D

E

7) Coincide con el Problema 10 del Nivel I.

8) Sea ABCD un cuadrilatero tal que AB = 3 cm, BC = 4 cm, CD = 13 cm y

AD = 12 cm. Si ∠ABC es recto, calcula el area, en cm2, de ABCD.

A

B

C

D

9) En una escuela hay 8 alumnos que desean formar equipos de tres. ¿Cuantos equipos

se pueden formar si se permite que dos equipos tengan a lo mas un alumno en

comun?

10) En una competencia internacional de matematicas, el 28% de los concursantes son

de Asia, el 10% de Oceanıa. Los concursantes de Africa junto con los de Europa

son el 40% del total, ademas Asia tiene 66 alumnos mas que los alumnos de Africa

y entre alumnos de Europa y de Oceanıa hay 187 alumnos. ¿Cuantos concursantes

europeos participaron?

11) Sea ABCD un rectangulo con diagonal AC, sea Q un punto sobre BC tal que

∠BAQ = ∠QAD y ∠QAC = 15. Encuentra la medida en grados del angulo

∠BOQ, donde O es el punto medio de AC.

12) Encuentra el mayor entero positivo n, tal que n2+2018n sea un cuadrado perfecto.

Page 48: TZALOA Revista de la Olimpiada Mexicana de Matematicas ... · de un tiempo, Sissa, le presento´ a su rey el juego que le hab´ıa pedido: el ajedrez. Lue-go de entender el juego

40 2a OMMEB, Concurso Nacional 2018

Nivel II. Parte B.

1) Muestra que el siguiente numero

4

3+

6

5+

8

7+ · · ·+ 102

101,

no es un numero entero.

2) En cada una de las 10 regiones en que se ha dividido el cırculo de la figura se colocan

3 fichas. Un movimiento consiste en mover una ficha a una region vecina (es decir,

a una region que comparte un radio). ¿Es posible que despues de 2018 movimientos

todas las fichas se encuentren en la misma region? Justifica tu respuesta.

3) Sea ABCD un paralelogramo y sean E un punto sobre AB tal que los angulos

∠ADE y ∠EDB son iguales, F la interseccion de DE con BC y G la interseccion

de AD con CE. Muestra que BC2 = BF ·AG.

B

A

C

D

E

F

G

Nivel III. Parte A.

1) Coincide con el Problema 5 del Nivel II.

2) Coincide con el Problema 8 del Nivel II.

3) Coincide con el Problema 9 del Nivel II.

4) Coincide con el Problema 10 del Nivel II.

5) Coincide con el Problema 11 del Nivel II.

6) Coincide con el Problema 12 del Nivel II.

7) La coleccion de numeros an se define como sigue:

a1 = 1 y an+1 =2an

2 + 3an, para n ≥ 1.

Page 49: TZALOA Revista de la Olimpiada Mexicana de Matematicas ... · de un tiempo, Sissa, le presento´ a su rey el juego que le hab´ıa pedido: el ajedrez. Lue-go de entender el juego

2a OMMEB, Concurso Nacional 2018 41

Encuentra el valor numerico de a67.

8) Sea ABC un triangulo isosceles cuyo angulo en A mide 24, siendo este el angulo

desigual. Un punto D en la circunferencia de centro C y radio AC es tal que BDinterseca al segmento AC. La perpendicular a BC por D corta a la circunferencia

en E. Encuentra ∠ADB + ∠BEA.

9) Lupita quiere invitarle un helado a cada uno de sus amigos Hugo, Ricardo y Deeds.

Para ello tiene tres conos y 7 bolas de helado para repartir: 2 de chocolate, 2 de

vainilla, 2 de fresa y 1 de limon. ¿De cuantas maneras puede formar y repartir los

helados, si usa las 7 bolas y cada uno de sus amigos debe tener un numero distinto

(positivo) de bolas en su helado? Nota: Las bolas del mismo sabor son identicas

entre sı, pero el orden en que se distribuyen las bolas en un cono sı importa. Por

ejemplo, los siguientes dos helados son distintos.

V

V

V

VF

F

10) Sea P un polıgono regular de n lados y vertices V1, V2, . . . , Vn, y sea O su centro.

Determina todos los posibles valores de n para que la bisectriz de ∠V2V1O pase por

V3.

11) Una lancha cuando se desplaza en un rıo tranquilo va a 9 km/h. Un dıa que habıa

corriente en el rıo, Jose recorrio un kilometro de ida y un kilometro de regreso en

15 minutos. ¿Cual era la velocidad, en km/h, de la corriente del rıo ese dıa?

12) En la siguiente figura, ACDE es un rectangulo y se han dibujado la circunferencia

inscrita al trianguloAFE y su diametro paralelo al lado FE. Encuentra la longitud,

en cm, de AB.

4 cm

7 cm

45

A

B

C

D EF

b

Nivel III. Parte B.

1) Coincide con el Problema 2 del Nivel II, Parte B.

Page 50: TZALOA Revista de la Olimpiada Mexicana de Matematicas ... · de un tiempo, Sissa, le presento´ a su rey el juego que le hab´ıa pedido: el ajedrez. Lue-go de entender el juego

42 2a OMMEB, Concurso Nacional 2018

2) Ana tiene cuatro hermanas: Berta, Ceci, Diana y Elena. Su edad actual es un numero

impar menor que 30. Cuando Berta tenga el triple de la edad actual de Ana, se

cumpliran las siguientes relaciones:

a) La suma de las edades que tendran en ese entonces Ana y Ceci sera igual a la

suma de las edades actuales de todas las hermanas.

b) La edad de Diana sera el triple de su edad actual.

c) La edad de Elena sera un ano mas que el doble de la edad actual de Berta.

Halla la suma de las edades de Ana y Berta.

3) En la figura, el sector AOB representa una cuarta parte de un cırculo de radio r = 1y el punto C satisface que ∠BOC = 45. Sea P un punto sobre el segmento OB(distinto de O y de B). Se trazan los segmentos AP y CP para formar la region

sombreada. Demuestra que el area de la region sombreada es menor que el area de

la region sin sombrear.

A

B

C

OP

Pruebas por Equipos

Nivel I.

1) Ordena los siguientes numeros de menor a mayor: 36, 45, 54, 63.

2) En un hexagono regular ABCDEF de area 1 cm2, se han trazado en su interior

tres triangulos congruentes ABP , CDQ y EFR con angulos de 30, 60 y 90,

los angulos rectos en P,Q,R, como se muestra en la figura. Encuentra el area, en

cm2, del triangulo PQR.

A

B C

D

EF

R

P

Q

3) Se acomodan 7 de los numeros del 1 al 8 en las caras de la siguiente figura, de forma

que para cada tres caras que se toquen en un mismo cırculo la suma de los numeros

en tales caras sea un multiplo de 3. ¿Cuales numeros podrıan sobrar en estos tipos

de acomodos?

Page 51: TZALOA Revista de la Olimpiada Mexicana de Matematicas ... · de un tiempo, Sissa, le presento´ a su rey el juego que le hab´ıa pedido: el ajedrez. Lue-go de entender el juego

2a OMMEB, Concurso Nacional 2018 43

4) Sergio y Zael quieren ir a una heladerıa a comprar un tipo de helado cada dıa de la

semana. Dentro de los artıculos que se venden se encuentran los siguientes: paletas,

raspados y sandwich de nieve. Ademas, de cada uno de los artıculos hay 4 sabores:

vainilla, fresa, chocolate y limon. Sergio quiere comprar un artıculo de chocolate

por dıa de manera que no coma lo mismo dos dıas seguidos, mientras que Zael

quiere comprar paletas de distintos sabores sin comer dos dıas seguidos el mismo

sabor. ¿Quien de los dos tiene mas formas distintas de comprar a lo largo de toda la

semana? Justifica tu respuesta.

5) Alguien cambio las etiquetas de los numeros de la calculadora de Cesar. Los nume-

ros deberıan estar en la posicion que muestra la imagen de la izquierda, pero sus

posiciones fueron cambiadas a como se muestra en la imagen de la derecha.

7 8 94 5 61 2 3

9 8 76 5 43 2 1

Como consecuencia de esto, cuando Cesar aprieta el numero 1, la calculadora re-

gistra el numero 3 y al reves. Lo mismo pasa con el 4 y con el 6 y con el 7 y el 9.

¿Cuantas multiplicaciones distintas de dos numeros de un solo dıgito, daran un re-

sultado incorrecto cuando Cesar utilice su calculadora? (Nota: las multiplicaciones

1× 2 y 2× 1 son consideradas multiplicaciones diferentes).

6) Encuentra el entero positivo mas pequeno de seis dıgitos, que cumpla que la suma

de sus seis dıgitos sea igual al producto de sus dıgitos.

7) Acomoda ocho numeros enteros diferentes en los cuadritos que faltan, de manera

que los productos de los tres numeros de cada renglon, de cada columna y de cada

diagonal sean iguales.

6

8) Se quiere acomodar 8 piezas como las de las derecha (las puedes rotar de ser ne-

cesario) de manera que se cubra toda la figura de la izquierda. ¿Cuantos acomodos

diferentes se pueden hacer?

Page 52: TZALOA Revista de la Olimpiada Mexicana de Matematicas ... · de un tiempo, Sissa, le presento´ a su rey el juego que le hab´ıa pedido: el ajedrez. Lue-go de entender el juego

44 2a OMMEB, Concurso Nacional 2018

Nivel II.

1) Coincide con el Problema 3 del Nivel I.

2) Coincide con el Problema 6 del Nivel I.

3) Encuentra todas las parejas de numeros reales (x, y) que cumplen las siguientes dos

igualdades: x3 + y3 = 1 y x2 + y2 = 1.

4) Encuentra todas las parejas de enteros positivos (a, r) tales que el numero N =a2 + (a+ r)2 + (a+2r)2 + (a+3r)2 +(a+4r)2 tenga todos sus dıgitos iguales.

5) Un triangulo ABC con vertices sobre una circunferencia de centro O tiene la

siguiente propiedad: si O, C′ son simetricos con respecto a C, se cumple que

∠CC′A = ∠ABC. Encuentra el valor (en grados) del angulo ∠ABC.

6) Coincide con el Problema 8 del Nivel I.

7) Los numeros creativos son numeros de 4 dıgitos abcd tales que los numeros de

dos dgitos ab y cd son ambos pares. Ademas, la suma de sus dıgitos es un numero

primo. Por ejemplo, 2018 es numero creativo, ya que ab = 20 y cd = 18 son

numeros pares de dos dıgitos y la suma 2 + 0 + 1 + 8 = 11 es un numero primo.

¿Cuantos numeros creativos menores o iguales que 2018 hay?

8) Sea ABCD un cuadrilatero, como se indica en la figura. Muestra que si los cua-

tro triangulos ABC, BCD, CDA, DAB, tienen el mismo perımetro, entonces

ABCD es un rectangulo.

AD

B C

Nivel III.

1) Sea A = 2, 5, 8, 11, . . . , 2018, cada numero, a partir del segundo, es el anterior

mas 3. Determina el mınimo valor k tal que si escogemos k numeros del conjunto

A, necesariamente hay dos distintos cuya suma sea 2020.

Page 53: TZALOA Revista de la Olimpiada Mexicana de Matematicas ... · de un tiempo, Sissa, le presento´ a su rey el juego que le hab´ıa pedido: el ajedrez. Lue-go de entender el juego

2a OMMEB, Concurso Nacional 2018 45

2) Todos los numeros impares se dividen en grupos como se indica:

1, 3, 5, 7, 9, 11, 13, 15, 17, 19, . . . .

¿Cual es la suma de los elementos del decimo grupo?

3) Coincide con el Problema 5 del Nivel II.

4) Coincide con el Problema 4 del Nivel II.

5) Coincide con el Problema 7 del Nivel II.

6) Coincide con el Problema 8 del Nivel II.

7) Consideramos un tablero de 8×8. El Batab es una pieza que puede moverse de una

casilla a otra vecina (que comparte un lado). Un camino del Mayab es un camino

que va de una casilla inicial a una final tal que:

a) Consta exclusivamente de movimientos del Batab.

b) En cada paso se aleja del punto inicial y se acerca al punto final.

Se coloca una ficha verde en una casilla y una ficha naranja en otra distinta, luego se

coloca una ficha blanca en una casilla que esta dentro de un camino del Mayab que

va de la ficha verde a la ficha naranja. Llamamos T al numero total de caminos del

Mayab que van de la ficha verde a la naranja pasando por la ficha blanca. Encuentra

el numero total de formas distintas en que se pueden colocar las tres fichas de modo

que 49 divida a T .

8) Los gemelos Adan y Beto van de su casa a la escuela. Adan, corre la mitad del

trayecto y camina la otra mitad, mientras que Beto corre la mitad del tiempo y

camina la otra mitad del tiempo. Los dos corren a una misma velocidad v1 y los

dos caminan a una misma velocidad v2. ¿Quien de ellos llega primero? Justifica tu

respuesta.

Soluciones de las Pruebas Individuales

Nivel I.

1) Tenemos que en 4 dıas 6 impresoras hacen 100 libros, por lo que en un dıa 6 im-

presoras hacen 1004 = 25 libros. Luego, en un dıa una impresora hace 25

6 libros.

De donde en un dıa 4 impresoras hacen 4 · 256 = 503 libros. Ası que en tres dıas 4

impresoras hacen 3 · 503 = 50 libros.

2) Cada tres cuadritos hay exactamente un cuadrito gris. Como 2018 = 672 · 3 + 2entonces tenemos un total de 672 + 1 = 673 cuadritos grises.

Page 54: TZALOA Revista de la Olimpiada Mexicana de Matematicas ... · de un tiempo, Sissa, le presento´ a su rey el juego que le hab´ıa pedido: el ajedrez. Lue-go de entender el juego

46 2a OMMEB, Concurso Nacional 2018

3) El numero de ninas en el club debe ser un multiplo de 8, es decir uno de los numeros

en la lista: 8, 16, 24, 32. De igual manera, el numero de ninos en el club debe ser

un multiplo de 7, es decir uno de los numeros en la lista: 7, 14, 21, 28, 35. Como

debe haber en total 37 estudiantes, debemos buscar dos numeros, uno en cada lista,

de tal forma que sumen 37. Esto se logra con 16 y 21. Por lo que hay 16 ninas en el

club.

4) Sean a y b numeros yucatecos, con a > b. Entonces para hacer la diferencia a−b lo

menor posible, lo mejor serıa que fueran diferentes unicamente en el numero de las

unidades, pero esto no es posible. Ası que estamos buscando numeros a y b que solo

difieran en los dıgitos de las unidades y decenas, ya que la diferencia entre esos dos

numeros serıa igual a la diferencia entre los numeros formados por los dıgitos de sus

decenas y los dıgitos de sus unidades. Un ejemplo de esto serıa tomar los numeros

32 y 23, y formar los numeros a = 987654132 y b = 987654123 de manera que

a− b = 32− 23 = 9. Ahora debemos asegurarnos que este es el mınimo. Para esto

observamos que la suma de los dıgitos de cualquier numero yucateco es igual a 45,

por lo que a y b son ambos multiplos de 9. Entonces, su diferencia a− b debera ser

un multiplo de 9, sin embargo como a 6= b, tenemos que a − b 6= 0. Por lo que el

mınimo valor que puede tomar a− b es 9.

5) El primer dıa gasta 19+1 pesos, el segundo dıa 19+2 y ası, el dıa 17 gasta 19+17y el dıa 18 le quedan 19 + 18 pesos, que se los gasta. Entonces tiene originalmente

(19 + 1) + (19 + 2) + (19 + 3) + · · ·+ (19 + 18) = 19 · 18 + 19·182 = 513 pesos.

6) Notemos que todos los numeros de la lista 1, 2, 3, . . . , 50 tienen al menos tres multi-

plos entre 101 y 200, y por tanto no son numeros mayas. De manera similar, los

numeros de la lista 51, 52, . . . , 100 al multiplicarlos por 2 caen entre 101 y 200.

Por tanto, todos ellos tienen al menos un multiplo en la lista. Como 66 · 3 = 198,

entonces todos los numeros del 51 al 66 tienen al menos dos multiplos en la lista y

por tanto no son numeros mayas. Dado que 67 · 3 = 201, podemos concluir que el

unico multiplo de 67 entre 101 y 200 es 67 × 2 = 114. Ası, concluimos que 67 es

el numero maya mas pequeno.

7) El perımetro del triangulo mayor es 96 cm y esta formado por 24 palillos que son

lados de los triangulos mas pequenos. Por lo tanto cada palillo pequeno mide 4 cm.

De ahı podemos obtener que el perımetro de cada triangulo pequeno es 12 cm. Si

contamos los triangulos pequenos podemos ver que son 27, por lo que la longitud

total de los palillos es 27× 12 = 324 cm.

Otra forma: En la figura hay 4 tamanos de triangulos. El mas grande tiene perımetro

96 cm, los siguientes disminuyen en tamano a la mitad, ası tendran perımetros 48cm, 24 cm y 12 cm, respectivamente. Del grande al menor hay 1, 1, 3 y 9 triangulos

de cada tamano. Luego, la suma de los perımetros es 96+48+3 ·24+9 ·12 = 324cm.

8) Necesitamos que al sumar 1, el denominador sea multiplo de 7, ası que el primer

candidato posible es la fraccion equivalente a 18 que tiene numerador 6, esto es

648 . Probamos con este candidato y podemos ver que al sumar 1 al numerador y al

denominador obtenemos la fraccion 749 , que al ser simplificada es 1

7 .

Page 55: TZALOA Revista de la Olimpiada Mexicana de Matematicas ... · de un tiempo, Sissa, le presento´ a su rey el juego que le hab´ıa pedido: el ajedrez. Lue-go de entender el juego

2a OMMEB, Concurso Nacional 2018 47

Otra forma: Buscamos a y b tales que ab = 1

8 y a+1b+1 = 1

7 . Esto es equivalente al

sistema 8a = b y 7a+ 7 = b+ 1, cuyas soluciones son a = 6 y b = 8 · 6 = 48.

9) Si E es el punto medio de BC, se tiene que BE = EC = AD. Luego, AD y

EC son segmentos paralelos y de la misma longitud, por lo que CDAE es un

paralelogramo y como tiene tres lados iguales entonces es un rombo.

A

B C

D

E

Ademas, el triangulo ABE es equilatero y entonces ∠ABE = 60 = ∠DCE. Co-

mo CA es bisectriz de ∠DCE (pues los triangulosADC y CEA son congruentes),

se tiene que ∠ACE = 30, por lo que ∠CAB = 90.

10) Consideremos la siguiente figura y notemos que el triangulo ABC tiene el mis-

mo perımetro que el hexagono. Mas aun, el area del triangulo es 46 del area del

hexagono. Por lo tanto, Area (ABC) = 46 · 120 = 80 cm2.

A

B C

11) La palabra YUCATAN tiene siete letras, de modo que podemos escoger de 27 − 2formas los distintos conjuntos de letras que pudieron haber quedado. Sin embargo,

algunas palabras estan representadas por dos de estos conjuntos. Estas palabras son

las que contienen exactamente una A y no tienen la letra T; esto es, las que podemos

formar con una A y las letras YUCN. Restando las repetidas, entonces el resultado

es (27 − 2)− 24 = 110.

Otra forma. Hay tres tipos de palabras que pueden quedar escritas. Las que no

tienen letra A, las que tienen exactamente una letra A y las que tiene dos letras A.

Con dos letras A, hay 25 − 1 palabras, ya que las otras letras estan o no, y una

menos porque no quedaron todas las letras. Ademas. hay 25 − 1 palabras que no

tienen letra A: las otras 5 letras pueden o no estar y una menos que corresponde

al caso en que se cayeron todas las letras. Con exactamente una A y sin la letra

T, hay 24 = 16, con la letra T y con la A antes de la T hay tambien 24 = 16 y

con la letra T y con A despues de la T, hay 24 = 16. Por lo tanto, hay en total

(25 − 1) + (25 − 1) + 16 + 16 + 16 = 110 palabras.

12) En total hay(

64

)

cuadrilateros con vertices sobre los puntos marcados. De estos, uno

tiene solo puntos blancos en su interior y otro tiene solo puntos grises. De tal modo,

hay(

64

)

− 2 = 15− 2 = 13 cuadrilateros bicolores.

Page 56: TZALOA Revista de la Olimpiada Mexicana de Matematicas ... · de un tiempo, Sissa, le presento´ a su rey el juego que le hab´ıa pedido: el ajedrez. Lue-go de entender el juego

48 2a OMMEB, Concurso Nacional 2018

13) Si hay A alumnos, entonces hubo 3A parejas que se formaron para bailar, y si B es

el numero de alumnas, se formaron 6B parejas de baile. Como 3A = 6B, se tiene

que A = 2B y como A+B = 90, se tiene que A = 60 y B = 30.

14) Supongamos que la longitud de los lados mayores de los rectangulos mas pequenos

es igual a y y la longitud de sus lados menores es x, mientras que la longitud del lado

menor del rectagulo mediano es igual a a y su lado mayor vale 2x. Como los lados

de los rectangulos pequenos y grande, estan en la misma proporcion tenemos queyx = a+y

2x . Por lo tanto, 2y2x = a+y

2x , de donde se concluye que a = y. Finalmente,

como el rectangulo pequeno y el rectangulo mediano tienen la misma proporcion se

obtiene que yx = 2x

y = 2y/x , por lo que ( yx)

2 = 2 y yx =√2.

15) En cada uno de los numeros que escribe Hugo, el 1 que aparece al principio siempre

es positivo. Para cada eleccion de signos podemos considerar la eleccion de signos

opuesta y al sumar estos numeros el resultado sera 1 + 1 = 2. Como hay 29 elec-

ciones de signos que se agrupan por pares y como cada par suma 2, se tendra que la

suma es 2 · (29/2) = 29.

Nivel II. Parte A.

1) Notemos que

732 − 312 − 91 = (73 + 31)(73− 31)− 91 = (23 · 13)(2 · 3 · 7)− 7 · 13= 7 · 13(24 · 3− 1) = 7 · 13 · 47.

Por lo tanto, son 3 primos los que dividen a 732 − 312 − 91.

2) Notemos que AB′ED es un cuadrilatero con ∠B′ = ∠D = 90, ademas AB′ =AD = 1 y DE = EB′ =

√2− 1. Luego,

Area(AB′ED) = Area(ADE) + Area(AB′E)

= 2Area(AB′E) = 2AB′ · EB′

2=√2− 1 =

1√2 + 1

.

3) Coincide con la solucion del Problema 13 del Nivel I.

4) Coincide con la solucion del Problema 14 del Nivel I.

5) Como un dado tiene 6 numeros, en total se pueden formar 6 × 6 = 62 numeros de

dos cifras. Por lo tanto, el total de casos (tomando en cuenta tanto los tiros de Isaac

como de Alfredo) es 62 × 62 = 64. Por otro lado, notemos que en 62 casos Isaac y

Alfredo obtienen los mismos resultados. Luego, en 64− 62 casos los resultados son

distintos y, de ellos, la mitad corresponden al caso en que el numero de Alfredo es

mayor. Por lo tanto, la probabilidad buscada es igual a(64−62)/2

64 = 12 − 1

72 = 3572 .

6) Como AE es paralelo a BD, el triangulo AEC es rectangulo. Si F es la intersec-

cion de AE con BC, se tiene que los triangulos rectangulos ABF y CEF son

Page 57: TZALOA Revista de la Olimpiada Mexicana de Matematicas ... · de un tiempo, Sissa, le presento´ a su rey el juego que le hab´ıa pedido: el ajedrez. Lue-go de entender el juego

2a OMMEB, Concurso Nacional 2018 49

semejantes por tener angulos en F iguales por ser opuestos por el vertice. Lue-

go, α = ∠BAE = ∠BAF = ∠FCE = ∠BCE y como BDC ∼ ABC,

entonces α = ∠BCE = ∠BCA. De lo anterior tenemos que 2α = ∠BAC y

α = ∠BCA. Por lo tanto, α = 30 y ∠CAB = 60.

A

B C

D

E

F

α

αα

7) Coincide con la solucion del Problema 10 del Nivel I.

8) Por el teorema de Pitagoras,AC =√AB2 +BC2 = 5. Por otro lado, notemos que

AD2 + AC2 = 122 + 52 = 132 = DC2. Entonces, por el recıproco del teorema

de Pitagoras tenemos que ADC es un triangulo rectangulo y ∠DAC es recto. Por

lo tanto, [ABCD] = [ABC] + [ADC] = 3 · 42 + 5 · 122 = 36 cm2.

9) Si 1, 2, 3, 4, 5, 6, 7, 8 son los alumnos, se pueden formar 8 equipos ası: (1, 2, 3),(1, 4, 5), (1, 6, 7), (2, 4, 6), (2, 5, 8), (3, 4, 8), (3, 5, 7), (6, 7, 8).Si A1, . . . , An son los equipos, entonces |Aj | = 3 con 1 ≤ j ≤ 8 y |Ai∩Aj | ≤ 1 si

i 6= j. Si existe a que este en cuatro distintas Ai y como solo hay un alumno comun

en dos equipos, los otros 8 alumnos que se necesitan para los 4 equipos deben ser

diferentes, por lo que debera haber al menos 1 + 2 · 4 = 9 alumnos, lo cual no es

posible. Por lo que un alumno pertenecera a lo mas a 3 equipos. Luego, con los 8alumnos podemos formar a lo mas 8·3

3 equipos, es decir, el numero n de equipos

debe cumplir que n ≤ 8·33 = 8.

10) Sea T el total de concursantes. De Asia hayA = 28100T , de Oceanıa hayO = 10

100T y

entre africanos y europeos hay Af+E = 40100T . Tambien se tiene que A = Af+66

y E +O = 187, por lo que 40100T = Af + E = (A− 66) + (187−O) = 28

100T −66 + 187− 10

100T . Luego, 40100T = 18

100T + 121, de donde T = 122 (12100) = 550.

Ası, a la competencia asistieron 550 alumnos. De Oceanıa asistieron 55 alumnos

que corresponden al 10% y, como E +O = 187, se tiene que de Europa asistieron

187− 55 = 132 competidores.

11) Denotemos por P a la interseccion de AQ y BO.

A

B C

D

O

Q

P

45

60

Page 58: TZALOA Revista de la Olimpiada Mexicana de Matematicas ... · de un tiempo, Sissa, le presento´ a su rey el juego que le hab´ıa pedido: el ajedrez. Lue-go de entender el juego

50 2a OMMEB, Concurso Nacional 2018

Como ∠BAQ = 45 y ∠QAC = 15 se tiene que ∠BAO = 60 y como Oes punto de interseccion de las diagonales, ∠OBC = ∠BCO = ∠OAD = 30,

luego ∠ABO = 60, por lo que el triangulo ABO es equilatero. Luego, ∠APB =180 − 60 − 45 = 75.

Como ABQ es un triangulo rectangulo isosceles con AB = BQ y como ABOes equilatero, se tiene que BO = AB = BQ. Luego, OBQ es isosceles y como

∠OBQ = 30 se tiene que ∠BOQ = ∠BQO = 75.

12) Sea m tal que n2 + 2018n = (n+m)2. Desarrollando y simplificando obtenemos

que n = m2

2018−2m . Notemos que se trata de una funcion creciente en m para 1 ≤m ≤ 1008. Ademas la expresion no esta definida para m = 1009 y n < 0 si

m ≥ 1010. Ası que el maximo se alcanza en m = 1008. Entonces, n = 10082

2 .

Nivel II. Parte B.

1) Notemos que

N =4

3+

6

5+ · · ·+ 102

101= (

3

3+

5

5+ · · ·+ 101

101)+ (

1

3+

1

5+ · · ·+ 1

101) = 50+B

Por lo tanto, N es un numero entero si y solo si el numero B = 13 + 1

5 + · · ·+ 1101

es entero. Sea C = 3 · 5 · 7 · · · · · 99. Es claro que si B es entero, entonces BC es

entero. Pero, BC = C3 + C

5 + · · · + C99 + C

101 es entero si y solo si C101 es entero,

pero esto ultimo es falso ya que 101 es primo y en la factorizacion en primos de Cel numero 101 no esta presente.

2) La respuesta es no. Supongamos que sı es posible hacerlo, y pintemos de negro la

region correspondiente. Coloreamos las nueve regiones restantes alternadamente de

gris y blanco como se muestra en la figura.

Observamos que si una ficha se encuentra en una region pintada de blanco, entonces

requirira de un numero impar de movimientos para llegar a la region roja, en tanto

que una ficha ubicada en una region gris ocupara un numero par de movimientos.

De ese modo, hay 3 × 5 = 15 fichas que requeriran cada una un numero impar de

movimientos para llegar a la region negra, haciendo un total impar de movimientos.

Por otro lado, las fichas de las casillas grises requieren un total par de movimientos.

Ası, el numero de movimientos para llegar a la configuracion deseada debe ser

necesariamente impar y, por lo tanto, no puede ser 2018.

3) Notemos que los triangulos BCE y AGE son semejantes, al igual que los triangu-

los BEF y AED, por lo que, BCAG = BE

AE y BEAE = BF

AD . Luego,BC·AD = BF ·AGy, como AD = BC, se tiene que BC2 = BF ·AG.

Page 59: TZALOA Revista de la Olimpiada Mexicana de Matematicas ... · de un tiempo, Sissa, le presento´ a su rey el juego que le hab´ıa pedido: el ajedrez. Lue-go de entender el juego

2a OMMEB, Concurso Nacional 2018 51

Nivel III. Parte A.

1) Coincide con la solucion del Problema 5 del Nivel II.

2) Coincide con la solucion del Problema 8 del Nivel II.

3) Coincide con la solucion del Problema 9 del Nivel II.

4) Coincide con la solucion del Problema 10 del Nivel II.

5) Coincide con la solucion del Problema 11 del Nivel II.

6) Coincide con la solucion del Problema 12 del Nivel II.

7) La formula de recursion se reescribe ası

1

an+1=

2 + 3an2an

=1

an+

3

2.

Si bn = 1an

, entonces bn+1 = bn + 32 = bn−1 +

32 · 2 = · · · = b1 +

32 · n. Luego,

b67 = b1 +3266 = 1 + 3(33) = 100 por lo que a67 = 1

100 .

8) La recta BC corta a la circunferencia en F con B entre F y C. Notemos que el

triangulo FDE es isosceles y, por lo tanto, ∠FDB = ∠FEB.

A

B C

D

E

F

Luego, ∠ADB +∠BEA = ∠ADB −∠FDB +∠BEA+∠FEB = ∠ADF +∠AEF = 2∠ADF = ∠ACF = 180−24

2 = 78.

9) Primero, notemos que las diferentes formas de escribir a 7 como la suma de tres

enteros positivos son: 5 + 1 + 1, 4 + 2 + 1, 3 + 3 + 1 y 3 + 2 + 2. De estas, la

unica que tiene tres sumandos distintos es 4 + 2 + 1. Si ponemos todas las bolas

de helado juntas se forma una ”palabra” de longitud 7 con dos C’s, dos V ’s, dos

F ’s y una L. Usando permutaciones con repeticion vemos que hay 7!2!2!2!1 = 630 de

estas palabras. Ahora bien, una palabra da origen a una forma de hacer 3 helados,

simplemente dividiendola en segmentos de longitud 4, 2 y 1 (ver figura).

Page 60: TZALOA Revista de la Olimpiada Mexicana de Matematicas ... · de un tiempo, Sissa, le presento´ a su rey el juego que le hab´ıa pedido: el ajedrez. Lue-go de entender el juego

52 2a OMMEB, Concurso Nacional 2018

Finalmente, solo basta permutar estos helados entre Hugo, Ricardo y Deeds para

obtener todas las formas requeridas. La respuesta es 3!× 630 = 3780.

10) Por ser P regular, el angulo interno ∠V2V1Vn mide, en grados,(n−2)180

n .

OVn

V1

V2

V3

V4

Como V1O es la bisectriz de ∠V2V1Vn, se tiene que ∠V2V1O = (n−2)90

n . Por

la hipotesis se tiene que ∠V2V1V3 = 12∠V2V1O = (n−2)45

n . Como el segmento

V1V3 es perpendicular al segmento V2O, y ∠V2OV1 = 360

n , por ser angulo central;

∠OV1V3 + ∠V2OV1 = 90. Sustituimos por los valores de ∠OV1V3 y ∠V2OV1,

esto es,(n−2)45

n + 360

n = 90. Multiplicando la ecuacion anterior por n45 se llega

a (n− 2) + 8 = 2n, con lo cual n = 6.

Otra forma. Consideremos el cuadrilatero OV1V2V3 que se forma de unir los dos

triangulos isosceles congruentes OV1V2 y OV2V3 . Es claro que OV2 es perpendi-

cular a V1V3 y que los triangulos V1V2V3 y OV1V3 son triangulos isosceles. Sea

P la interseccion de OV2 y V1V3, los cuatro triangulos PV1V2, PV3V2, PV3O,

PV1O, son congruentes (por criterio ALA). Luego, OV1V2V3 es un rombo y en

consecuencia OV1V2 es un triangulo equilatero, por lo que ∠V1OV2 = 60. Pero

por otro lado, ∠V1OV2 = 360n , por ser angulo central del polıgono regular de n

lados. Igualando los valores de ∠V1OV2 obtenemos que n = 6.

11) Si v es la velocidad de la corriente, la lancha avanza a (9 + v) km/h cuando va a

favor de la corriente y a (9 − v) km/h cuando va en contra de la corriente. Si t1es el tiempo que tarda cuando va con la corriente a su favor, entonces se cumple

que (9 + v)t1 = 1. Y si t2 es el tiempo que tarda en recorrer el kilometro cuando

va contra corriente entonces (9 − v)t2 = 1. Nos dicen que t1 + t2 = 14h, luego

14 = t1+t2 = 1

9+v+1

9−v . Esto es equivalente a 1892−v2 = 1

4 , es decir 4·18 = 81−v2,

Page 61: TZALOA Revista de la Olimpiada Mexicana de Matematicas ... · de un tiempo, Sissa, le presento´ a su rey el juego que le hab´ıa pedido: el ajedrez. Lue-go de entender el juego

2a OMMEB, Concurso Nacional 2018 53

por lo que v2 = 81− 72 = 9 y entonces v = 3 km/h.

12) Denotemos por r y s a los valores del inradio y el semiperımetro del triangulo

AFE, respectivamente. Tenemos que Area(AFE) = b×h2 = 3·4

2 = 6. Por otro

lado, Area (AFE) = s ·r =(

3+4+52

)

·r = 6r. Luego, 6r = 6, de donde obtenemos

que r = 1.

4 cm

7 cm

45

A

B

C

D EF A′

F ′

E′

Ib

b

b

b

Siendo A′ y F ′ como en la figura e I el incentro del triangulo AFE, IF ′EA′

forma un cuadrado de lado r = 1, por lo que EF ′ = 1 y, por el teorema de Tales,

se concluye que ABAF = AF ′

F ′E , de donde AB = AF ′·AFF ′E = 3·5

4 = 154 .

Nivel III. Parte B.

1) Coincide con el Problema 2 del Nivel II, Parte B.

2) Sea x la cantidad de anos entre el presente y el momento en el que Berta tenga el

triple de la edad actual de Ana. Entonces, los enunciados se traducen en el siguiente

sistema de ecuaciones:

B + x = 3A, (2)

A+ x+ C + x = A+B + C +D + E, (3)

D + x = 3D, (4)

E + x = 2B + 1. (5)

De la ecuacion (3) podemos ver que x = 2D. Sustituyendo este valor en las ecua-

ciones (2) y (4) y resolviendo el sistema resultante obtenemos que D = 3B+15 y

E = 4B+35 . Sustituyendo x = 2D = 6B+2

5 en la ecuacion (1) obtenemos que

15A = 11B + 2. Ademas, como A es impar entonces 11B + 2 debe terminar en 5,

por lo que B debe terminar en 3. Por otro lado, como A ≤ 29 entonces tenemos la

desigualdad

B =15A− 2

11≤

15 · 29− 2

11

= 39,

ası que B = 3, 13, 23, 33. Probando estos casos, verificamos que la unica solucion

entera es B = 23 y A = 17. Por lo tanto, la suma de las edades de Ana y Berta es

23 + 17 = 40.

Nota: Tambien se puede resolver la ecuacion diofantina por los metodos usuales y

encontrar que B = 23 es la unica solucion impar con 1 ≤ A ≤ 29.

Page 62: TZALOA Revista de la Olimpiada Mexicana de Matematicas ... · de un tiempo, Sissa, le presento´ a su rey el juego que le hab´ıa pedido: el ajedrez. Lue-go de entender el juego

54 2a OMMEB, Concurso Nacional 2018

3) Sean x = OP y Q el pie de la altura de C sobre OB.

A

B

C

OPx Q

Notemos que la region sin sombrear esta compuesta de dos partes. Area(OAP ) =r · x2 = x

2 . El area de la segunda parte se puede calcular restando al area del sector

COB el area del triangulo OCP . Ahora bien, el triangulo OCQ es rectangulo

isosceles de hipotenusa r = 1, ası que CQ = 1√2

. Por lo tanto,

Area(sector COB) − Area(OCP ) =π

8− x(1/

√2)

2=

π

8− x

2√2.

De este modo, tenemos la desigualdad x2 + π

8 − x2√2= π

8 +√2−12 x > π

8 . En otras

palabras, la region sin sombrear tiene siempre un area mayor a la mitad del area del

sector OAB. En consecuencia, la region sombreada siempre tendra menor area que

la region sin sombrear.

Otra forma. Sea R la interseccion de AP y OC. Es claro que Area(AOP ) >Area(COP ) ya que estos triangulos tienen por base comun a OP , y la altura desde

A es mayor a la altura desde P . Como el triangulo ROP es comun a estos dos

triangulos, se tiene que Area(AOR) > Area(CRP ). Por lo tanto,

Area(region sombreada) < Area(sectorAOC) ≤ Area(region sin sombrear).

Soluciones de las Pruebas por Equipos

Nivel I.

1) Como 63 = 216, 54 = 625, 36 = 729 y 45 = (22)5 = 210 = 1024, tenemos que

63 < 54 < 36 < 45.

2) Notemos que el triangulo PQR es una cuarta parte del triangulo AEC. Ademas,

el triangulo AEC tiene la mitad del area del hexagono. Por lo tanto, el area del

triangulo PQR es 14 × 1

2 = 18 del area del hexagono. Ası, Area(PQR) = 1

8 .

3) Los numeros son 3 y 6. Si una cara no central tiene un numero a multiplo de 3,

entonces si d esta en una cara a dos caras de distancia de a (sin pasar por el centro),

estos tienen dos vecinos en comun (como se muestra en la figura). Si estas tienen

los numeros b y c, entonces a+ b+ c es multiplo de 3 y b+ c+ d es multiplo de 3,

de donde se sigue que su diferencia, igual a a− d, es multiplo de 3.

Page 63: TZALOA Revista de la Olimpiada Mexicana de Matematicas ... · de un tiempo, Sissa, le presento´ a su rey el juego que le hab´ıa pedido: el ajedrez. Lue-go de entender el juego

2a OMMEB, Concurso Nacional 2018 55

6 3

1 2

4

57

8

1 2

4

57

8c

a b

d

Como a es multiplo de 3, d tambien lo es. Entonces no puede haber un multiplo de

3 fuera del centro ya que habrıa al menos 3 caras con multiplos de 3 y, del 1 al 8,

hay 2 multiplos de 3. Concluimos que se puede tener a lo mas un multiplo de 3 y

como se omite solo un numero, este debe ser 3 o 6. Para ambos casos se encuentra

un acomodo que funciona.

4) Sergio tiene el primer dıa 3 posibilidades de pedir: paletas, raspados y sandwich de

nieve. Del dıa 2 al 7, tiene solo dos posibilidades en cada dıa, porque si un dıa pide

sandwich de nieve, al siguiente solo puede pedir paletas o raspados. Por el principio

del producto, Sergio tiene 3× 26 = 192 formas. Por otro lado, Zael tiene el primer

dıa 4 posibilidades de pedir y, del dıa 2 al 7, tiene solo 3 posibilidades en cada dıa,

por un argumento analogo. Por el principio del producto, Zael tiene 4× 36 = 2916formas. Por lo tanto Zael tiene mas formas.

5) Notemos que unicamente 6 numeros han sido cambiados de posicion, el 1 fue cam-

biado por el 3, el 4 por el 6 y el 7 por el 9, mientras que los numeros 2, 5 y 8 se

mantuvieron en su lugar original. Entonces, de las multiplicaciones de un numero

por el mismo, 3 son correctas y las otras 6 son equivocadas. A partir de ahora consi-

deraremos multiplicaciones de dos numeros diferentes. Empecemos analizando las

multiplicaciones donde ambos numeros son alguno de los numeros 2, 5, 8. Todas

estas seran correctas y hay 3× 2 = 6 de ellas. Ahora, si ambos numeros que teclea

Cesar son algunos de 1, 4, 7, entonces la calculadora registrara dos numeros entre 3,

6, 9, por lo que el resultado sera multiplo de 9 y, como ninguno de 1, 4, 7 es multi-

plo de 3, el resultado no sera correcto. De manera similar, si ambos numeros que

teclea Cesar son algunos de 3, 6, 9, el resultado deberıa de ser multiplo de 9, pero

como la calculadora registra algunos de los numeros 1, 4, 7, no lo sera. Es decir, en

todos estos casos el resultado sera incorrecto. El numero de posibilidades que hay

es (3× 2)+ (3× 2) = 12. Pasemos ahora al caso en que uno de los numeros es uno

de 2, 5, 8 y el otro es uno de 1, 3, 4, 6, 7, 9. En todos estos casos la multiplicacion

sera incorrecta. Hay (3× 6) + (6× 3) = 18 + 18 = 36 de estos casos. Finalmente

vemos que pasa cuando uno de los numeros es uno de 1, 4, 7 y el otro de 3, 6, 9.

Claramente las multiplicaciones 1 × 3, 3 × 1, 4 × 6, 6 × 4, 7 × 9 y 9 × 7 seran

correctas. Son 6 multiplicaciones correctas y las restantes 12 multiplicaciones seran

incorrectas. Por lo tanto, hay 6 + 12 + 36 + 12 = 66 multiplicaciones incorrectas.

6) Ningun dıgito debe ser cero y no puede tener 5 o 6 dıgitos iguales a 1, (ya que no

es posible que se cumpla: 5 + a = a o 6 = 1).

Si tiene 4 dıgitos iguales a 1, los otros dos dıgitos a y b cumplen que a+b+4 = ab,que es equivalente a que (a − 1)(b − 1) = 5. Luego, a = 6 y b = 2 o bien a = 2

Page 64: TZALOA Revista de la Olimpiada Mexicana de Matematicas ... · de un tiempo, Sissa, le presento´ a su rey el juego que le hab´ıa pedido: el ajedrez. Lue-go de entender el juego

56 2a OMMEB, Concurso Nacional 2018

y b = 6. Entonces, el numero que se busca es 111162 o 111126 y, el segundo es el

mas pequeno.

7) Sı es posible. Una manera es la siguiente. Se coloca el 1 a un lado de 6, independien-

te del numero x. En las esquinas a y b se deben colocar numeros que su producto

sea 6, estos son 2 y 3 (no pueden ser 1 y 6 porque los 9 numeros son diferentes).

a1 6 x

b

En las otras esquinas se colocan numeros que se ajusten para que el producto de los

numeros en las diagonales sean iguales. Digamos ası,

2 · 6 21 6

3 · 6 3

Como el producto es 63, lo que falta queda ası:

2 · 6 32 21 6 62

3 · 6 22 3

8) Si coloreamos la figura como tablero de ajedrez, de gris y blanco, notamos que una

pieza siempre debe cubrir dos triangulitos grises, o bien dos triangulitos blancos.

Contaremos las maneras de acomodar los triangulitos de cada color y bastara aplicar

el principio del producto. La region blanca se puede llenar con 3 piezas de 2 maneras

distintas (ver figura).

A

C

B B C

A

B BC

C A

A

La region gris se llena con 5 piezas de la siguiente manera: primero se coloca la

pieza que va en el vertice superior del triangulo –para ello hay dos casos– y se

observa que cada caso se completa de 3 maneras distintas (en la figura se ilustra

uno de esos casos). Ası, hay 2×3 = 6 maneras de llenar la region gris. Por lo tanto,

la respuesta es 2× 6 = 12.

Page 65: TZALOA Revista de la Olimpiada Mexicana de Matematicas ... · de un tiempo, Sissa, le presento´ a su rey el juego que le hab´ıa pedido: el ajedrez. Lue-go de entender el juego

2a OMMEB, Concurso Nacional 2018 57

A

C

B

B

C

A

B

B

C

C

A

A

D D

E E

E

ED D

B

B

C C

A

A

E

E

D

D

Nivel II

1) Coincide con la solucion del Problema 3 del Nivel I.

2) Coincide con la solucion del Problema 6 del Nivel I.

3) Notemos que (x, y) = (0, 1) y (x, y) = (1, 0) son soluciones. Por la segunda igual-

dad x y y tienen valor absoluto menor o igual que 1. Ademas, no existe solucion

con x = y, tampoco con x = −1 y tampoco con y = −1. Supongamos que x < yy analicemos los siguientes casos:

a) Si x > 0, entonces tenemos que 0 < x < y ≤ 1. Por lo tanto, x3 < x2 y

y3 < y2 y ası 1 = x3 + y3 < x2 + y2 = 1, lo cual es absurdo.

b) De manera similar, si x < 0 entonces y3 = 1 − x3 > 1, luego y > 1, contradi-

ciendo la segunda ecuacion.

Por lo tanto, la unica alternativa es x = 0 y entonces y = 1. Concluimos que las

unicas soluciones son (x, y) = (0, 1) y (x, y) = (1, 0).

4) Sea c = a+ 2r, entonces

N = (c− 2r)2 + (c− r)2 + c2 + (c+ r)2 + (c+ 2r)2 = 5c2 + 10r2

tiene todas su cifras iguales. Como N es divisible entre 5, acaba en 5 o 0. Luego,

todas las cifras de N deberan ser 0 o 5. El primer caso implica que N = 0 y

en consecuencia a = r = 0, lo cual es absurdo. En el segundo caso tenemos

que N5 = c2 + 2r2 tiene todas sus cifras iguales a 1. Por lo tanto, si N

5 tiene

tres cifras o mas, entonces N5 ≡ 111 ≡ 7 (mod 8). Sin embargo, dado que los

cuadrados son congruentes a 0, 1 o 4 modulo 8, entonces tenemos que c2 + 2k2

es congruente a 0, 1, 2, 4 o 6 modulo 8, lo cual es contradictorio. Por lo tanto,

tenemos que N5 = 1 o N

5 = 11. El primer caso implica que k = 0, por lo que

a = b = c = d = e = 1, llegando nuevamente a un absurdo. Finalmente, al

resolver c2 + 2k2 = 11 obtenemos que c = 3 y k = 1. De este modo, la unica

solucion es la progresion 1, 2, 3, 4, 5.

5) Sea β = ∠ABC. Consideremos a O′ el centro del circuncırculo del triangulo

ACC′. Por la medida del angulo inscrito, ∠CO′A = 2∠CC′A = 2∠ABC = 2β.

Tenemos tambien por ser isosceles los triangulos AO′C y AOC, que ∠O′AC =∠ACO′ = 90−β = ∠OAC = ∠OCA, por lo que son congruentes los triangulos

AO′C y AOC. Luego, O′C = O′A = OA = OC. Como ∠OCO′ = 180 − 2β,

Page 66: TZALOA Revista de la Olimpiada Mexicana de Matematicas ... · de un tiempo, Sissa, le presento´ a su rey el juego que le hab´ıa pedido: el ajedrez. Lue-go de entender el juego

58 2a OMMEB, Concurso Nacional 2018

se tiene que ∠O′CC′ = 180 − ∠OCO′ = 2β y O′C = OC = CC′ y, como

O′C = O′C′ (ya que O′ es el centro del circuncırculo de ACC′), se tiene que

CC′O′ es un triangulo equilatero, de donde ∠O′CC′ = 2β = 60, por lo que

β = 30.

6) Coincide con la solucion del Problema 8 del Nivel I.

7) Haremos una demostracion por casos. Como abcd < 2108, tenemos que a = 1 o

a = 2. Cuando a = 1 entonces tenemos la ecuacion

b+ c+ d = p− 1.

Si p = 2, entonces la unica solucion es b = d = 0, c = 1, por lo que el unico

numero creativo en este caso es 1010. Si p es un primo impar, dado que b y d son

pares, por paridad concluimos que c tambien lo es. Ademas, como cd es un numero

de dos cifras tenemos que c 6= 0. Por lo tanto, podemos hacer el cambio de variables

b = 2B, c = 2(C + 1), d = 2D y transformar la ecuacion anterior en

B + C +D =p− 3

2

sujeta a las condiciones 0 ≤ B ≤ 4, 0 ≤ C ≤ 3, 0 ≤ D ≤ 4. Por lo tanto, tenemos

quep− 3

2≤ 4 + 3 + 4 = 11 y en consecuencia p ≤ 25. La siguiente tabla ilustra

los valores correspondientes a los primos p que cumplen la desigualdad.

p 3 5 7 11 13 17 19 23p−32 0 1 2 4 5 7 8 10

En cada caso se puede resolver la ecuacion usando el metodo de separadores y el

principio de inclusion-exclusion. La siguiente tabla muestra todas las posibilidades:

p−32 Soluciones Total

0(

22

)

1

1(

32

)

3

2(

42

)

6

4(

62

)

−(

22

)

14

5(

72

)

−[(

32

)

+ 2(

22

)]

16

7(

92

)

−[(

52

)

+ 2(

42

)]

14

8(

102

)

−[(

62

)

+ 2(

52

)]

10

10(

122

)

−[(

82

)

+ 2(

72

)]

+[

2(

32

)

+(

22

)]

3

Ası que hay un total de 67 soluciones en este caso. Finalmente, consideremos el

caso a = 2 tenemos que p debe ser impar y las unicas posibilidades son 2010,

2012, 2014 y 2018. Hemos probado ası que hay 1+67+4 = 72 numeros creativos.

Page 67: TZALOA Revista de la Olimpiada Mexicana de Matematicas ... · de un tiempo, Sissa, le presento´ a su rey el juego que le hab´ıa pedido: el ajedrez. Lue-go de entender el juego

2a OMMEB, Concurso Nacional 2018 59

8) Supongamos que a = AB, b = BC, c = CD, d = DA, e = AC y f = BD. Si

los perımetros de los triangulos son iguales, se tiene que a+ b+ e = b + c+ f =c+ d+ e = d+ a+ f . Luego,

a+ e = c+ fb+ f = d+ ec+ e = a+ fb+ e = d+ f

de donde, a− c = f − e = d − b = e − f . Ahora, como f − e = e − f , se tiene

que e = f . Luego, a = c, f = e y d = b. Pero un cuadrilatero con lados opuestos

iguales y con las diagonales iguales, tiene que ser un rectangulo.

Nivel III

1) Notemos primero queA = 3n+2 | 0 ≤ n ≤ 672. Queremos asegurar que existen

n y m tales que 3n+ 2 + 3m+ 2 = 2020, es decir n+m = 672. Agrupamos los

numeros del 0 al 672, en los conjuntos

0, 672, 1, 671, . . . , 335, 337, 336.

Notemos que cada uno de ellos, excepto el conjunto 336 es de la forma n, 672−n, garantizando ası que si escogemos dos numeros del mismo conjunto, hallamos

la pareja (3n+2, 3(672−n)+2)cuya suma es 2020. Por el principio de las casillas,

necesitarıamos al menos 338 numeros para garantizar que esto pasa. Entonces k =338.

2) Despues de hacer algunos casos, surge la conjetura que la suma de los elementos

del n-esimo grupo es n3. Para demostrarlo, notemos que el n-esimo grupo esta

conformado por los los impares desde 2(

n(n−1)2

)

+1 hasta 2(

n(n+1)2

)

−1. Como

la suma 1+ 3+5+ · · ·+2k− 1 es igual a k2, entonces tenemos que los elementos

del n-esimo bloque suman

[

n(n+ 1)

2

]2

−[

n(n− 1)

2

]2

=(n4 + 2n3 + n2)− (n4 − 2n3 + n2)

4= n3.

Por tanto, la respuesta es 103 = 1000.

3) Coincide con la solucion del Problema 5 del Nivel II.

4) Coincide con la solucion del Problema 4 del Nivel II.

5) Coincide con la solucion del Problema 7 del Nivel II.

6) Coincide con la solucion del Problema 8 del Nivel II.

7) Observemos primero que el numero de caminos del Mayab que van de una casilla xa una casilla y es igual a

(

m+nm

)

=(

m+nn

)

donde m es el numero de casillas que hay

que recorrer en direccion horizontal para ir de x a y, y n las que hay que recorrer en

Page 68: TZALOA Revista de la Olimpiada Mexicana de Matematicas ... · de un tiempo, Sissa, le presento´ a su rey el juego que le hab´ıa pedido: el ajedrez. Lue-go de entender el juego

60 2a OMMEB, Concurso Nacional 2018

direccion vertical. Entonces tenemos que T =(

a+ba

)(

c+dc

)

donde el primer factor

corresponde al recorrido de la ficha verde a la blanca, y el segundo al recorrido de

la ficha blanca a la naranja. El camino del Mayab mas largo, correspondiente a ir

de una esquina del tablero a la opuesta en diagonal, tiene 14 movimientos. Como la

ficha blanca no puede estar en una esquina se tiene que a+b ≤ 13 y c+d ≤ 13, y se

sigue que en los coeficientes binomiales considerados a lo mucho hay un factor 7,

por lo que 49 no puede dividir a ninguno de los dos. Por lo tanto, 7 divide a ambos.

Ambos recorridos deben tener entonces al menos 7 movimientos y, como la suma

de ambos no puede exceder a 14, por fuerza a+ b = 7 = c + d. Esto corresponde

a que el camino del Mayab de la verde a la naranja tiene 14 movimientos y, por

lo tanto, ambas fichas deben estar en esquinas opuestas del tablero. El valor de ay el de c es cualquier numero del 1 al 6, lo que significa que la ficha blanca puede

estar en cualquier casilla del interior del tablero (que no este en las orillas). Con esto,

obtenemos que el numero de formas en que T sea un multiplo de 49 es 4 ·36 = 144,

que corresponden a 4 formas de elegir la esquina verde y 36 de colocar la esquina

blanca.

8) Sean d la distancia entre la casa y la escuela, v1 la velocidad en que corren y v2 la

velocidad en que caminan. Si Adan tarda t1 horas en correr la mitad del trayecto

y t2 horas en caminar la otra mitad, se tiene que d2 = v1t1 = v2t2. Por lo que su

recorrido lo hace en el tiempo

t1 + t2 =d

2v1+

d

2v2=

d

2

(

v1 + v2v1v2

)

.

Por otro lado, Beto se mueve con diferentes velocidades en tiempos iguales. Si tarda

t horas en hacer el recorrido, entonces

d = v1t

2+ v2

t

2, por lo que t =

2d

v1 + v2.

Ahora comparemos los tiempos en que tardan en llegar,

t1 + t2t

=

d2

(

v1+v2v1v2

)

1v1+v2

=(v1 + v2)

2

4v1v2.

Pero v1v2 ≤(

v1+v22

)2(por la desigualdad MG −MA), por lo que t1+t2

t ≥ 1,

y entonces t1 + t2 ≥ t, pero a menos que v1 = v2, se tiene que t1 + t2 > t; en

consecuencia Beto llega primero.

Page 69: TZALOA Revista de la Olimpiada Mexicana de Matematicas ... · de un tiempo, Sissa, le presento´ a su rey el juego que le hab´ıa pedido: el ajedrez. Lue-go de entender el juego

32a Olimpiada Mexicana de

Matematicas

Concurso Nacional

Del 4 al 9 de noviembre de 2018 se llevo a cabo en Campeche, Campeche, el Concurso

Nacional de la 32a Olimpiada Mexicana de Matematicas, con la participacion de todos

los estados del paıs.

Los 16 alumnos ganadores del primer lugar (ordenados por estados) fueron:

Sofıa Ingigerth Canas Urbina (Chiapas).

Fabian Domınguez Lopez (Chiapas).

Tomas Franciso Cantu Rodrıguez (Ciudad de Mexico).

Diego Hinojosa Tellez (Ciudad de Mexico).

Ana Paula Jimenez Dıaz (Ciudad de Mexico).

Nuria Sydykova Mendez (Ciudad de Mexico).

Bruno Gutierrez Chavez (Colima).

Isaac Pancardo Botello (Guanajuato).

Jesus Omar Sistos Barron (Guanajuato).

Rigoberto Concepcion Rodrıguez Cruz (Hidalgo).

Jonatan Alejandro Gonzalez Cazares (Jalisco).

Eric Ivan Hernandez Palacios (Nuevo Leon).

Pablo Alhui Valeriano Quiroz (Nuevo Leon).

Carlos Alberto Paez De la Cruz (Queretaro).

Ivan Garcıa Mestiza (Veracruz).

Ricardo de Jesus Balam Ek (Yucatan).

Los 11 alumnos preseleccionados para la Olimpiada Matematica de Centroamerica y

el Caribe fueron:

Page 70: TZALOA Revista de la Olimpiada Mexicana de Matematicas ... · de un tiempo, Sissa, le presento´ a su rey el juego que le hab´ıa pedido: el ajedrez. Lue-go de entender el juego

62 Concurso Nacional 2018, 32a OMM

Kevin Brian Rodrıguez Sanchez (Baja California).

Leornardo Mikel Cervantes Mateos (Ciudad de Mexico).

Ana Illanes Martınez de la Vega (Ciudad de Mexico).

Omar Farid Astudillo Marban (Guerrero).

Luis Eduardo Martınez Aguirre (Nuevo Leon).

David Garcıa Maldonado (Oaxaca).

Saul Villalobos Fajardo (Oaxaca).

Monica Isabel Casillas Rodrıguez (Queretaro).

Karla Rebeca Munguıa Romero (Sinaloa).

Daniel Alejandro Ochoa Quintero (Tamaulipas).

Jacobo de Juan Millon (Yucatan).

Las 12 alumnas preseleccionadas para la Olimpiada Europea Femenil de Matematicas

fueron:

Sofıa Ingigerth Canas Urbina (Chiapas).

Katia Garcıa Orozco (Chihuahua).

Mirena Flores Valdez (Ciudad de Mexico).

Ana Paula Jimenez Dıaz (Ciudad de Mexico).

Ana Illanes Martınez de la Vega (Ciudad de Mexico).

Nuria Sydykova Mendez (Ciudad de Mexico).

Nathalia del Carmen Jasso Vera (Guanajuato).

Ana Paula Ramırez Sanchez (Jalisco).

Laura Itzel Rodrıguez Dimayuga (Morelos).

Monica Isabel Casillas Rodrıguez (Queretaro).

Karla Rebeca Munguıa Romero (Sinaloa).

Ana Teresa Calderon Juarez (Zacatecas).

Aunque la participacion en el Concurso Nacional es individual, es importante destacar

la labor que han llevado los estados de la Republica apoyando a sus concursantes.

Con el proposito de reconocer este trabajo, presentamos el registro de los estados que

obtuvieron los primeros 10 lugares en el Concurso Nacional de la 32a OMM.

1. Ciudad de Mexico.

2. Guanajuato

3. Nuevo Leon.

4. Jalisco.

5. Sinaloa.

6. Yucatan.

7. Chihuahua.

8. Chiapas.

9. Veracruz.

10. San Luis Potosı.

En esta ocasion, el premio a la Superacion Academica se llamo “Copa San Francisco

de Campeche” y fue ganado por Guanajuato. El segundo y tercer lugar de este premio

lo ocuparon Sinaloa y Veracruz, respectivamente.

Page 71: TZALOA Revista de la Olimpiada Mexicana de Matematicas ... · de un tiempo, Sissa, le presento´ a su rey el juego que le hab´ıa pedido: el ajedrez. Lue-go de entender el juego

Concurso Nacional 2018, 32a OMM 63

A continuacion presentamos los problemas y soluciones del 32 Concurso Nacional de

la OMM. Los alumnos tuvieron dos sesiones de cuatro horas y media cada una para

resolverlos.

Problema 1. Sean A y B dos puntos en una recta ℓ, M el punto medio del segmento

AB y X un punto del segmento AB, diferente de M . Sea Ω una semicircunferencia de

diametro AB. Considera un punto P sobre Ω y considera Γ la circunferencia tangente

a AB que pasa por P y por X . Sea Q la otra interseccion de Γ con Ω. La bisectriz

del angulo ∠PXQ interseca a Γ en un punto R. Sea Y un punto en ℓ, tal que RY es

perpendicular a ℓ. Muestra que MX > XY .

(Problema sugerido por Victor Domınguez Silva)

Solucion. Notemos que R es el punto medio del arco PQ de Γ que no contiene a X .

Sea O el centro de Γ. Veamos que P y Q son las intersecciones de la circunferencia

de diametro AB, de la cual claramente M es centro, con Γ. Entonces R,O,M son

colineales por estar todos en la mediatriz de PQ. Luego, como el segmento RY es

paralelo al segmento OX y este ultimo es perpendicular a AB, por el Teorema de

Tales, YXXM = RO

OM < 1, pues OR es igual al radio de Γ y OM es mayor a este por

encontrarse M fuera de Γ. De aquı obtenemos la desigualdad deseada.

b

A

b

B

bM

b

X

bO

bP

bQ

bR

b

Y

Problema 2. Para cada entero positivo m, la figura Lm se forma traslapando dos

rectangulos, uno de m× 1 y uno de 1 ×m de manera que coincida un cuadrito extre-

mo del primero con un cuadrito extremo del segundo, como se muestra en la siguiente

imagen.

. . .

L1 L2 L3 L4

Page 72: TZALOA Revista de la Olimpiada Mexicana de Matematicas ... · de un tiempo, Sissa, le presento´ a su rey el juego que le hab´ıa pedido: el ajedrez. Lue-go de entender el juego

64 Concurso Nacional 2018, 32a OMM

Usando algunas figuras Lm1, Lm2

, . . . , Lmk, se cubre completamente una cuadrıcula

de n×n, colocandolas de manera que sus bordes esten sobre las lıneas de la cuadrıcula.

De entre todas las posibles formas de cubrir la cuadrıcula, con distintos valores para

los mi y para k, determina el mınimo valor posible de m1 +m2 + · · ·+mk.

Nota: Para cubrir la cuadrıcula, las figuras pueden reflejarse, rotarse, traslaparse o

salirse de la cuadrıcula.

(Problema sugerido por Leonardo Ariel Garcıa Moran)

Solucion de Isaac Pancardo Botello. Primero demostraremos que k ≥ n. Suponga-

mos, por contradiccion, que k < n. Como cada ficha tiene exactamente una esquina y

hay n filas en el tablero de n × n, hay al menos una fila que no contiene una esquina

sobre ella. Luego, cada ficha tiene a lo mas un cuadrito sobre dicha fila y, como hay ncuadritos en la fila, son necesarias para llenarla por lo menos n fichas, lo que es una

contradiccion ya que k < n. Por lo tanto, k ≥ n.

Por otro lado, como cada ficha Lmitiene 2mi − 1 cuadritos y es posible cubrir la

cuadrıcula de n× n usando estas fichas, tenemos que (2m1 +1)+ (2m2 +1)+ · · ·+(2mk − 1) ≥ n2, esto es, 2(m1 +m2 + · · ·+mk) ≥ n2 + k y, como k ≥ n, se sigue

que n2 + k ≥ n2 + n. Por lo tanto, m1 +m2 + · · ·+mk ≥ n2+n2 = n(n+1)

2 .

Para concluir que el valor mınimo de m1+m2+· · ·+mk esn(n+1)

2 , solo hace falta dar

un acomodo con las fichas. Para esto, colocamos las n fichas compartiendo esquinas

con una diagonal y ordenadas de la mas grande a la mas pequena, colocandolas como

se muestra en la figura.

b

bb

b

Problema 3. Una sucesion a1, a2, . . . , an de enteros positivos se dice campechana, si

para cada i tal que 2 ≤ i ≤ n, se tiene que exactamente ai elementos de la suce-

sion son primos relativos con i. Decimos que el tamano de la sucesion es n − 1. Sea

m = p1p2 · · · pk donde p1, p2, . . . , pk son numeros primos distintos y k ≥ 2. Demues-

tra que existen al menos dos sucesiones campechanas de tamano m.

(Problema sugerido por Jorge Fernandez Hidalgo y Victor Hugo Almendra Hernandez)

Solucion de Carlos Alberto Paez de la Cruz. Primero veamos que existe al menos

una sucesion campechana de longitud m para m = p1 · p2 . . . pk, es decir, m es el

producto de primos distintos. Construyamos una sucesion campechana de la siguiente

manera: Sea di el maximo comun divisor entre i,m. Entonces hacemos ai =mdi

. Vea-

mos que dicha sucesion es campechana.

Page 73: TZALOA Revista de la Olimpiada Mexicana de Matematicas ... · de un tiempo, Sissa, le presento´ a su rey el juego que le hab´ıa pedido: el ajedrez. Lue-go de entender el juego

Concurso Nacional 2018, 32a OMM 65

Sea x un entero con 2 ≤ x ≤ n. Contemos la cantidad de numeros en la sucesion

que son primos relativos con x. Queremos llegar a que esa cantidad es md , donde

d = (m,x), pues ası ax = md y la cantidad de primos relativos con x serıa ax.

Sea x = (qα1

1 · qα2

2 . . . qαl

l )(rβ1

1 · rβ2

2 . . . rβss ), donde cada qi, ri es primo y, ademas,

qi | m para todo i y, ri ∤ m para todo i. Es claro que d = q1 . . . ql, por defini-

cion de los qi’s. Para que un numero ay sea primo relativo con x, debe cumplirse que

mcd(ay, qi) = 1 para todo i y que mcd(ay, ri) = 1 para todo i. Es claro que todo ayen la sucesion cumple la segunda condicion, pues ai =

mdi

es un divisor de m y, como

mcd(ri,m) = 1, debe ocurrir que mcd(ay, ri) = 1. Entonces solo es de importancia la

primera condicion: mcd(ay , qi) = 1. Como ay = mdy

y dy es el mınimo comun divisor

entre y y m, se sigue que si qi | y, entonces qi ∤mdy

(porque qi | y y qi | m implican

que qi | dy , pero la maxima potencia de qi que divide a m es q1i ). En otras palabras, si

qi | y entonces mcd(ay, qi) = 1.

Ahora veamos que si mcd(ay , qi) = 1, entonces qi | y. Tenemos que mcd(ay, qi) = 1es equivalente a mcd(m

dy, qi) = 1. Pero qi | m, por lo tanto qi | dy para que sea posible.

Luego, qi | dy = mcd(y,m). Entonces, qi | y.

Concluimos que si mcd(ay, qi) = 1, entonces qi | y. En otras palabras, si qi es un pri-

mo que divide a x y a m, el numero ay sera primo relativo con qi si y solo si qi divide

a y. Ademas, para que ay sea primo relativo con x, debe pasar que mcd(ay, qi) = 1para todo primo qi que divida a x y a m. Entonces, qi | y para todo i. Ademas, como

el maximo comun divisor de x y m es el producto de todos los qi’s, y estos son distin-

tos entre sı, ocurre que mcd(ay, qi) = 1 si y solo si d | y. Como hay exactamente md

multiplos de d entre 2 y m + 1, es claro que la cantidad de primos relativos con x es

igual a md = ax por definicion.

Para terminar el problema, se debe hallar otra sucesion campechana de longitud m.

Para ello, utilizaremos el hecho de que m tiene al menos 2 factores primos. Definimos

la sucesion de la siguiente forma: sea (nuevamente) di el maximo comun divisor de me i y sea:

ai =

mdi

si p1 ∤ i y p2 ∤ i, o p1 | i y p2 | i;

mdi· p1

p2si p1 | i y p2 ∤ i;

mdi· p2

p1si p1 ∤ i y p2 | i.

Veamos que esa sucesion tambien es campechana. La idea clave es ver que la sucesion

es similar a la que ya probamos que lo es. Vayamos por casos:

i) 2 ≤ x ≤ m y (p1 | xy p2 x) o (p1 ∤ xy p2 ∤ x. Entonces, ax = mdx

. Comparemos

en la primera sucesion. El numero con subındice x es el mismo en la original que en

la nueva. Ademas, la cantidad de primos relativos con x en la original es igual a ax.

Ademas, habıamos visto que mcd(ay, x) = 1 si y solo si dx | y. Supongamos que p1y p2 dividen a x. Si en la sucesion original mcd(ay, x) = 1, quiere decir que tanto p1como p2 dividen a y. Como p1 | y, p2 | y, en la nueva sucesion ax se mantiene igual. Si

mcd(ay, x) > 1, al multiplicar ay por p2

p1o por p1

p2o al dejarlo igual, mcd(a′y, x) seguira

siendo mayor que 1. Es decir, si p1 | x y p2 | x, la cantidad de primos relativos con

x seguira siendo mdx

. Ahora supongamos que p1 ∤ x y m ∤ x. En la sucesion original,

Page 74: TZALOA Revista de la Olimpiada Mexicana de Matematicas ... · de un tiempo, Sissa, le presento´ a su rey el juego que le hab´ıa pedido: el ajedrez. Lue-go de entender el juego

66 Concurso Nacional 2018, 32a OMM

hay mdx

primos relativos con x. Ademas, al mantener el numero de la original ay en la

nueva, o multiplicarlo por p1

p2o por p2

p1, el numero mcd(a′y, x) no cambiara. Entonces

seguira habiendo mdx

primos relativos con x.

ii) 2 ≤ x ≤ m y (p1 ∤ x, p2 ∤ x). Los numeros que estan en una posicion k con

(p1 | k, p2 | k) o (p1 ∤ k, p2 ∤ k) se mantendran igual con ambas sucesiones. Los que

estan en una posicion k con (p1 | k, p2 ∤ k) cambiaran a mdk· p1

p2. Ademas, los que

estaban en una posicion k con (p1 ∤ k, p2 | k) cambian a mdk· p2

p1. Sea ay un numero

en la original. Si mcd(ay, x) = 1, p1 | y y p2 ∤ y, entonces ay cambia a ay · p1

p2,

pero entonces a′y (el nuevo) ya no es primo relativo con x, pues p1 | x y p1 | ay · p1

p2.

En general, si p1 | k, p2 ∤ k y mcd(ay, p1) = 1, entonces mcd(a′y, p1) 6= 1, pero

mcd(a′y, p2) sı sera 1. En otras palabras, si en la original un numero era primo relativo

con p1, en la nueva sera primo relativo con p2, y viceversa.

Por lo tanto, la sucesion tendra mdi· p1

p2primos relativos con x por el cambio que se

hace entre p1 y p2. El otro caso con p1 ∤ x y p2 | x es analogo. Entonces, al cambiar la

sucesion original por la nueva, sigue siendo campechana y se concluye el problema.

Problema 4. Sea n ≥ 2 un numero entero. Para cualquier sucesion a1, a2, . . . , akde enteros positivos tales que a1 + a2 + · · · + ak = n, considera las sumas Si =1+2+ · · ·+ ai para 1 ≤ i ≤ k. Determina, en terminos de n, el maximo valor posible

del producto S1S2 · · ·Sk. (Problema sugerido por Angel Misael Pelayo Gomez)

Solucion. Sean a1, a2, . . . , ak una coleccion de enteros positivos (a la cual llamaremos

una configuracion valida) tales que a1+a2+· · ·+ak = n, de tal manera que el producto

M toma el valor maximo. Recordemos que Si = 1 + 2 + · · · + ai =ai·(ai+1)

2 , para

cada i, con 1 ≤ i ≤ k. La idea de la demostracion es ir cambiando ciertos numeros aisin modificar la suma original y ver como cambia el producto M .

Supongamos que existe ai ≥ 5, notemos que si en lugar de poner ai, utilizamos ak+1 =ai− 2 y ak+2 = 2, entonces a1+ a2 + · · ·+ ai−1 + ai+1+ · · ·+ ak + ak+1 + ak+2 =a1 + a2 + · · · + ak = n, pero el producto M cambia de M = S1 · S2 · · ·Sk a M ′ =S1 · S2 · · ·Si−1 · Si+1 · · ·Sk · Sk+1 · Sk+2, donde Sk+1 = 1 + 2 + · · · + (ai − 2) y

Sk+2 = 1 + 2 = 3. Veamos que M < M ′, para ello basta ver que Si < Sk+1 · Sk+2,

es decir, ai(ai+1) < (ai− 2)(ai− 1)(3), lo cual es equivalente a 2a2i − 10ai+6 ≥ 0.

Pero esta ultima desigualdad es verdadera, ya que 2a2i − 10ai ≥ 0 se sigue de que

ai ≥ 5. En resumen, si alguno de los numeros en la configuracion que nos genera un

producto M maximo, es mayor o igual que 5, lo podemos cambiar por dos numeros

que generan un valor de M mas grande. Por lo tanto ai ≤ 4, para toda 1 ≤ i ≤ k.

Ahora, supongamos que ai = 1 para algun i y, como n ≥ 2, existe al menos otro

aj , entonces podemos cambiar este ai = 1 por a′j = aj + 1, de esta manera la suma

de los numeros al sigue siendo n, pero M cambia de tener un factoraj(aj+1)

2 a un

factor(aj+1)(aj+2)

2 por lo que M se incrementa. Por lo tanto, 2 ≤ ai ≤ 4, para toda

1 ≤ i ≤ k.

Supongamos que existen i 6= j tales que ai = aj = 2, entonces estos dos numeros

los podemos sustituir por solo un numero a′i = 4, de esta forma la suma no cambia,

sigue siendo n, sin embargo cuando hay dos numeros dos, en M hay dos factores

iguales a 3, es decir un 9, que al cambiarlos por un numero 4, ahora aparece un factor

Page 75: TZALOA Revista de la Olimpiada Mexicana de Matematicas ... · de un tiempo, Sissa, le presento´ a su rey el juego que le hab´ıa pedido: el ajedrez. Lue-go de entender el juego

Concurso Nacional 2018, 32a OMM 67

1+2+3+4 = 10, por lo que M se incrementa. Luego, cada vez que aparezcan parejas

de numeros 2, los podemos cambiar por un 4 en la configuracion.

Supongamos que existen i 6= j tales que ai = aj = 4, entonces estos dos numeros

los podemos sustituir por dos numeros iguales a 3 y un numero igual a 2. De esta

forma la suma sigue siendo n. Pero originalmente en M aparecen dos factores iguales

1 + 2 + 3 + 4 = 10, es decir, un factor 100. Pero al cambiarlos por el par de numeros

3 y el numero 2, se tendra dos factores iguales a 1 + 2 + 3 = 6 y un factor igual a

1+ 2 = 3, es decir, 6× 6× 3 = 108 > 100, luego M crece. De esta manera, cada vez

que aparezcan parejas de numeros 4, los podemos cambiar por dos numeros 3 y un 2en la configuracion.

En el caso de algun ai = 4 y un aj = 2, los podemos cambiar por dos numeros 3. Ası

la suma sigue siendo n, pero nuevamente elproducto M pasa de tener un factor 3 y un

factor 10 a tener dos factores iguales a 6, pero 30 < 36, luego M vuelve a crecer.

En conclusion, en la configuracion de enteros positivos a1, a2, . . . , ak, con a1 + a2 +· · ·+ ak = n y M maximo, solo pueden aparecer numeros 3 y a lo mas un numero 2 o

un numero 4.

Para determinar el valor de M , analicemos modulo 3:

a) Si n ≡ 0 (mod 3), entonces n = 3s, para algun entero positivo s. Luego, todos los

ai son iguales a 3 y cada Si = 6, de donde M = 6s.

b) Si n ≡ 1 (mod 3), entonces n = 3s + 1, para algun entero positivo s, de donde

n = 3 + 3 + · · · + 3 + 4, donde hay s − 1 numeros iguales a 3 en la suma. Por lo

tanto, M = 10 · 6s−1.

c) Si n ≡ 2 (mod 3), entonces n = 3s + 2, para algun entero positivo s, de donde

n = 3+ 3+ · · ·+3+ 2, donde hay s numeros iguales a 3 en la suma. Por lo tanto,

M = 3 · 6s.

Problema 5. Sea n ≥ 5 un numero entero y considera un n-agono regular. Original-

mente, Nacho se encuentra en un vertice del n-agono, en el cual pondra una bandera.

El comenzara a moverse entre los vertices del n-agono, siempre en el sentido de las

manecillas del reloj. Primero se movera una posicion y colocara otra bandera, luego,

se movera dos posiciones y colocara otra bandera, etcetera, hasta que en el ultimo mo-

vimiento se movera n− 1 posiciones y colocara una bandera, de manera que colocara

n banderas en total. ¿Para que valores de n, Nacho colocara una bandera en cada uno

de los n vertices?

(Problema sugerido por Victor Domınguez Silva)

Solucion. Supongamos que Nacho no recorrio todos los vertices del n-agono. Esto

quiere decir que hay un vertice que Nacho recorrio al menos dos vertices. Considere-

mos uno de esos vertices y los dos primeros momentos que Nacho visito dicho vertice

y sean a el numero de posiciones que Nacho se movio en el movimiento que lo llevo

al vertice por primera vez (es posible que a sea igual a 0) y b el numero de posiciones

que Nacho se movio en el movimiento que lo llevo al vertice por segunda vez. Entre

Page 76: TZALOA Revista de la Olimpiada Mexicana de Matematicas ... · de un tiempo, Sissa, le presento´ a su rey el juego que le hab´ıa pedido: el ajedrez. Lue-go de entender el juego

68 Concurso Nacional 2018, 32a OMM

las dos visitas a este vertice Nacho se movio (a + 1) + (a + 2) + · · · + b posiciones.

Por lo tanto, n divide a la suma (a + 1) + (a + 2) + · · · + b, para ciertos enteros a, bcon 0 ≤ a < b ≤ n− 1. Usando la formula de Gauss, esta suma es igual a:

(a+ 1) + (a+ 2) + · · ·+ b = (1 + 2 + · · ·+ b)− (1 + 2 + · · ·+ a)

=b(b+ 1)− a(a+ 1)

2=

b2 + b− a2 − a

2

=(b − a)(a+ b+ 1)

2.

Por lo tanto, concluimos que Nacho habra estado dos veces en el mismo vertice si y

solo si 2n divide a (b−a)(a+ b+1) para ciertos enteros a y b con 0 ≤ a < b ≤ n− 1.

Supongamos primero que n es una potencia de 2, digamos, n = 2t con t un entero

positivo y que efectivamente se tiene que 2n = 2t+1 divide a (b − a)(a + b + 1) para

ciertos enteros a, b con 0 ≤ a < b ≤ n−1. Como estos factores tienen distinta paridad,

tenemos que 2n debe dividir a alguno de los factores b− a o a+ b+1, pero esto no es

posible pues 0 < b− a < a+ b+ 1 ≤ (n− 2) + (n− 1) + 1 = 2n− 2 < 2n, lo que

es una contradiccion. Concluimos que si n es una potencia de 2, Nacho recorrio los nvertices.

Demostraremos ahora que en el resto de los casos, Nacho no recorrio los n vertices.

Sea n = 2km donde m ≥ 3 es un numero impar y k ≥ 0. Queremos que 2n = 2k+1mdivida a (b − a)(a + b + 1). Una manera de que esto pase es que estos dos numeros

sean iguales: 2k+1m = (b − a)(a+ b+ 1).Consideraremos dos casos (mas adelante se vera la necesidad de separar en estos dos

casos).

a) m < 2k+1. Supongamos que m = b − a y 2k+1 = a + b + 1. Esto nos da el

sistema de ecuaciones b + a = 2k+1 − 1 y b − a = m, el cual tiene por solucion

a = 2k+1−m−12 y b = 2k+1+m−1

2 . Como 2k+1 es par y m es impar, estos valores de

a y b son enteros. Si verificamos que ademas se cumple que 0 ≤ a < b ≤ n − 1habremos terminado este caso.

0 ≤ a es equivalente a 2k+1 ≥ m + 1 lo cual es cierto (justo este valor de aes el que nos hace tener que dividir en estos casos).

a < b es cierto pues b− a = m > 0.

b ≤ n−1 se sigue de b ≤ a+b = 2(2k)−1 < 3(2k)−1 ≤ m(2k)−1 = n−1.

Esto concluye este caso.

b) m > 2k+1. En este caso supondremos que m = a+ b+1 y 2k+1 = b− a. Esto nos

da el sistema de ecuaciones b+a = m−1 y b−a = 2k+1, el cual tiene por solucion

a = m−2k+1−12 y b = m+2k+1−1

2 . Como 2k+1 es par y m es impar, estos valores de

a y b son enteros. Si verificamos que ademas se cumple que 0 ≤ a < b ≤ n − 1habremos terminado este caso.

0 ≤ a es equivalente a m ≥ 2k+1 + 1 lo cual es cierto.

a < b es cierto pues b− a = 2k+1 > 0.

Page 77: TZALOA Revista de la Olimpiada Mexicana de Matematicas ... · de un tiempo, Sissa, le presento´ a su rey el juego que le hab´ıa pedido: el ajedrez. Lue-go de entender el juego

Concurso Nacional 2018, 32a OMM 69

b ≤ n− 1 se sigue de b ≤ a+ b = m− 1 ≤ m2k − 1 = n− 1.

Esto concluye este caso y el problema.

Segunda solucion. Esta solucion se basa en que si s es un entero impar, la suma de una

cantidad s de enteros consecutivos es multiplo de s, de manera que si n tiene un factor

impar, podemos encontrar una suma adecuada en algunos casos (esto no funcionara

siempre pues a veces algunos sumandos son negativos).

Sea 2t la maxima potencia de 2 que divide a n. Si n = 2t, Nacho recorrera todos los

vertices (ver la primera solucion). Queda el caso n = 2t(2m+1) para m ≥ 1. Veremos

que podemos expresar a n como suma de algunos enteros consecutivos entre 1 y n− 1,

para ver que en este caso Nacho no recorrera todos los vertices. Consideremos la suma

(m − (2t − 1)) + · · · + (m − 1) + m + (m + 1) + · · · + (m + (2t − 1)) + (m +2t) = 2t+1m + 2t = 2t(2m + 1). Si m ≥ 2t, tenemos que 1 ≤ m − (2t − 1) y

m + 2t < 2t(2m + 1) = n y esto mostrarıa que para este valor de n, se repetira

un vertice. Si m ≤ 2t − 1 consideramos la suma (2t −m) + · · · + (2t − 1) + 2t +(2t + 1) + · · · + (2t +m) = 2t(2m + 1) que tambien funciona, pues 1 ≤ 2t −m y

2t +m < 2t(2m+ 1) = n.

Problema 6. Sean ABC un triangulo acutangulo y Γ la circunferencia que pasa por los

puntos A, B y C. La bisectriz del angulo en B corta a Γ en M y la bisectriz del angulo

en C corta a Γ en N . Sea I el punto de interseccion de las bisectrices anteriores. Con-

sidera M ′ y N ′ las reflexiones de M y N con respecto a CA y AB, respectivamente.

Muestra que el centro de la circunferencia que pasa por los puntos I , M ′ y N ′ esta en

la altura del triangulo ABC que pasa por A.

(Problema sugerido por Victor Domınguez Silva y Leonardo Ariel Garcıa Moran)

Solucion de Angel Alexis Anaya Alamea. Sean O1 el circuncentro del triangulo

ABC, D el pie de la altura desde A sobre BC y O2 el punto en AD tal que AO1 =AO2. Demostraremos que O2 es el circuncentro del triangulo IM ′N ′.

Como BM y CN son bisectrices de los angulos ∠CBA y ∠ACB, respectivamente,

tenemos que ∠ACN = ∠NCB = θ y ∠CBM = ∠MBA = β. Por angulos inscritos

que abren el mismo arco, tenemos que ∠ACN = ∠ABN = θ, ∠NV B = ∠NAB =θ, ∠CBM = ∠CAM = β y ∠MAB = ∠MCA = β. Lo anterior implica que los

triangulos ABN y CAM son isosceles, entonces NA = NB y MA = MC.

Sean K y L los puntos medios de AB y AC, respectivamente. Por lo desarrollado en el

parrafo anterior se tiene que KN y LM son mediatrices de los segmentos AB y AC,

respectivamente. Al ser N ′ la reflexion de N sobre AB, se tiene que NN ′ es perpen-

dicular a AB y NK = KN ′. Por lo tanto, las diagonales del cuadrilatero ANBN ′ se

bisecan, de donde se sigue que este cuadrilatero es un paralelogramo que de hecho es

un rombo. De manera analoga se concluye que AMCM ′ es un rombo.

Por otro lado, al ser O1 el circuncentro del triangulo ABC se tiene que los puntos N ,

K , N ′ y O1 estan alineados ası como M , L, M ′ y O1. De los paralelogramos se tiene

que ∠NAB = ∠N ′BA = θ, ∠ABN = ∠BAN ′ = θ, ∠CAM = ∠M ′CA = β y

∠MCA = ∠M ′AC = β.

Page 78: TZALOA Revista de la Olimpiada Mexicana de Matematicas ... · de un tiempo, Sissa, le presento´ a su rey el juego que le hab´ıa pedido: el ajedrez. Lue-go de entender el juego

70 Concurso Nacional 2018, 32a OMM

Ademas, es bien conocido2 que, al ser I el incentro del triangulo ABC y N la inter-

seccion de la bisectriz por C con el circuncırculo, se cumple que NA = NB = NI .

Lo anterior junto con el rombo implican que NI = AN ′.

bA

bB

bC

bL

bKbN

bM

bO1b

Ib

O2

b

M ′

b N′

Por la construccion de O2 y el hecho de que O1 es el circuncentro del triangulo ABC,

se tiene que NO1 = AO2. Por lo tanto, si demostramos que los angulos ∠INO2 y

∠N ′AO2 son iguales, se concluira por el criterio LAL que los triangulos NIO1 y

AN ′O2 son congruentes, de donde IO1 = N ′O2. Para ver esto notemos que

∠N ′AO2 = ∠N ′AB − ∠O2AB = θ − (90 − 2β) = θ + 2β − 90

mientras que

∠INO1 = ∠BNO − ∠BNI = 90 − θ − ∠BAC = 90 − θ − (180 − 2θ − 2β)

= θ + 2β − 90.

Lo anterior concluye la congruencia deseada. De manera similar obtenemos que IO1 =M ′O2. Por ultimo, es conocido que la altura desde A y el diametro por A (radio AO1)

son isogonales, de donde se tiene que ∠O2AI = ∠IAO1. Ademas, AO2 = AO1 por

construccion, luego por criterio LAL los triangulos AO2I y AO1I son congruentes, de

donde IO2 = IO1, lo cual implica que IO2 = M ′O2 = N ′O2 que nos dice que O2

es el circuncentro del triangulo IM ′N ′ el cual, por construccion, esta sobre la altura

desde A a BC, justo como se querıa.

2Ver en el apendice el teorema 17.

Page 79: TZALOA Revista de la Olimpiada Mexicana de Matematicas ... · de un tiempo, Sissa, le presento´ a su rey el juego que le hab´ıa pedido: el ajedrez. Lue-go de entender el juego

Apendice

Definicion 1 (Divisibilidad). Si a y b son enteros, se dice que a divide a b o que b es

multiplo de a si b = aq para algun entero q, y se denota por a | b.

Definicion 2 (Congruencias). Dados dos enteros a, b y un entero positivo m, decimos

que a es congruente con b modulo m si a−b es multiplo de m. En este caso escribimos

a ≡ b (mod m).

Teorema 1 (Propiedades de las congruencias). Sean a, b, c, d,m enteros con m ≥ 1.

1. Si a ≡ c (mod m) y c ≡ d (mod m), entonces a ≡ d (mod m).

2. Si a ≡ c (mod m) y b ≡ d (mod m), entonces ab ≡ cd (mod m).

3. Si a ≡ c (mod m), entonces an ≡ cn (mod m) para todo entero positivo n.

4. Si ab ≡ bc (mod m), entonces a ≡ c (mod m(b,m) ) donde (b,m) denota el

maximo comun divisor de b y m.

Teorema 2 (Pequeno de Fermat). Si p es un numero primo y a es un entero primo

relativo con p, entonces ap−1 ≡ 1 (mod p).

Teorema 3 (Induccion). El metodo de induccion se usa para demostrar que una pro-

posicion P (n) es verdadera para todo entero n ≥ k0, donde k0 es un entero fijo. El

metodo funciona de la siguiente manera:

1. Caso base: Se demuestra que P (k0) es verdadera.

2. Hipotesis de induccion: Se supone verdadera la proposicion P (k) para algun

entero k ≥ k0.

3. Se demuestra que P (k + 1) es verdadera.

Concluimos entonces que P (n) es verdadera para todo entero n ≥ k0.

Page 80: TZALOA Revista de la Olimpiada Mexicana de Matematicas ... · de un tiempo, Sissa, le presento´ a su rey el juego que le hab´ıa pedido: el ajedrez. Lue-go de entender el juego

72 Apendice

Teorema 4 (Combinaciones). Dado un conjunto A de n elementos, una combinacion

de m elementos de A, es un subconjunto de A formado de m elementos. El numero de

combinaciones de m elementos de A, denotado por(

nm

)

, es igual a

(

n

m

)

=n!

(n−m)!m!,

donde n! denota el producto 1 · 2 · · ·n.

Teorema 5 (Binomio). Para a y b numeros cualesquiera y n un entero no negativo se

cumple que

(a+ b)n =

n∑

k=0

(

n

k

)

akbn−k.

Teorema 6 (Desigualdad MA-MG: media aritmetica - media geometrica). Si x1, x2,

. . . , xn son numeros reales positivos, entonces

x1 + x2 + · · ·+ xn

n≥ n√x1x2 · · ·xn

y la igualdad se cumple si y solo si x1 = x2 = · · · = xn.

Teorema 7 (Suma de los angulos internos de un triangulo). La suma de los angulos

internos de un triangulo es 180.

Teorema 8 (Pitagoras). En un triangulo rectangulo, el cuadrado de la hipotenusa es

igual a la suma de los cuadrados de los catetos.

Definicion 3 (Congruencia de triangulos). Los triangulos ABC y A′B′C′ son con-

gruentes si los angulos y los lados del triangulo ABC son iguales a los angulos y los

lados del triangulo A′B′C′.

Criterio 1 (Criterio de congruencia LLL). Un criterio de congruencia de triangulos

nos dice que si tenemos dos triangulos con sus tres lados correspondientes iguales,

entonces son congruentes. A este criterio se le llama lado-lado-lado y lo denotamos

como LLL.

Criterio 2 (Criterio de congruencia ALA). Un criterio de congruencia de triangulos

nos dice que si tenemos dos triangulos con un lado igual y dos angulos adyacentes

iguales, entonces son congruentes. A este criterio se le conoce como angulo-lado-

angulo y lo denotamos como ALA.

Definicion 4 (Semejanza de triangulos). Los triangulos ABC y A′B′C′ son semejan-

tes, si sus angulos respectivos son iguales, es decir, ∠ABC = ∠A′B′C′, ∠ACB =∠A′C′B′ y ∠BAC = ∠B′A′C′; y sus lados homologos son proporcionales, esto esABA′B′

= BCB′C′

= CAC′A′

.

Criterio 3 (Criterio de semejanza AA). Si dos pares de angulos correspondientes de

los triangulos ABC y A′B′C′ son iguales, entonces los triangulos son semejantes. A

esta relacion le llamamos angulo-angulo y la denotamos como AA.

Page 81: TZALOA Revista de la Olimpiada Mexicana de Matematicas ... · de un tiempo, Sissa, le presento´ a su rey el juego que le hab´ıa pedido: el ajedrez. Lue-go de entender el juego

Apendice 73

Teorema 9 (Tales). Si ABC es un triangulo y D, E son puntos sobre los lados ABy CA, respectivamente, entonces los segmentos DE y BC son paralelos si y solo siABAD = AC

AE .

Teorema 10 (Bisectriz). Dado un triangulo ABC y un punto D sobre el lado BC, se

tiene que BDDC = BA

AC .

Teorema 11 (Ceva). Si L, M y N son puntos sobre los lados (o extensiones)BC,CA y

AB, respectivamente, del triangulo ABC, entonces AL, BM y CN son concurrentes

si y solo si BLLC · CM

MA · ANNB = 1.

Teorema 12 (Menelao). En un triangulo ABC, si L, M y N son puntos sobre los

lados BC, CA y AB, respectivamente (o sobre sus extensiones), entonces L, M y

N son colineales si y solo si BLLC · CM

MA · ANNB = −1, donde los segmentos se estan

considerando como segmentos dirigidos.

Definicion 5 (Angulos en la circunferencia).

1. Angulo inscrito. Es el angulo formado por dos cuerdas que comparten un punto

comun.

2. Angulo seminscrito. Es el angulo formado por una cuerda y la tangente a la

circunferencia en un punto comun.

3. Angulo central. Es el angulo formado por dos radios.

Teorema 13 (Medida del angulo inscrito). La medida de un angulo inscrito en una

circunferencia es igual a la mitad del angulo central que abre el mismo arco.

Teorema 14 (Medida del angulo seminscrito). La medida de un angulo seminscrito en

una circunferencia es igual a la mitad del angulo central que abre el mismo arco.

Teorema 15 (Potencia de un punto).

1. Si dos cuerdas AB y CD de una circunferencia se intersectan en un punto P ,

entonces PA · PB = PC · PD.

2. Si A, B y T son puntos sobre una circunferencia y la tangente en T intersecta

en un punto P a la prolongacion de la cuerda AB, entonces PT 2 = PA · PB.

Definicion 6 (Cuadrilatero cıclico). Un cuadrilatero es cıclico si sus cuatro vertices

estan sobre una misma circunferencia.

Teorema 16 (Cuadrilatero cıclico). Un cuadrilatero convexo ABCD es cıclico si y

solo si la suma de los angulos opuestos es igual a 180, esto es, ∠DAB + ∠BCD =∠ABC + ∠CDA = 180.

Teorema 17 (Circuncırculo e Incentro). Si Ω es el circuncırculo de un trianguloABC,

I es el incentro y M es la interseccion de AI con Ω, entonces MI = MB = MC.

Page 82: TZALOA Revista de la Olimpiada Mexicana de Matematicas ... · de un tiempo, Sissa, le presento´ a su rey el juego que le hab´ıa pedido: el ajedrez. Lue-go de entender el juego

Bibliografıa

[1] A. Alberro Semerena, R. Bulajich Manfrino, C. J. Rubio Barrios. Problemas

avanzados de olimpiada. Cuadernos de Olimpiadas de Matematicas. Instituto de

Matematicas de la UNAM, 2010.

[2] R. Bulajich Manfrino, J. A. Gomez Ortega, R. Valdez Delgado. Desigualdades.

Cuadernos de Olimpiadas de Matematicas. Instituto de Matematicas, UNAM,

2010.

[3] R. Bulajich Manfrino, J. A. Gomez Ortega, R. Valdez Delgado. Algebra. Cuader-

nos de Olimpiadas de Matematicas. Instituto de Matematicas, UNAM, 2016.

[4] R. Bulajich Manfrino, J. A. Gomez Ortega. Geometrıa. Cuadernos de Olimpiadas

de Matematicas. Instituto de Matematicas de la UNAM, 2010.

[5] R. Bulajich Manfrino, J. A. Gomez Ortega. Geometrıa. Ejercicios y Problemas.

Cuadernos de Olimpiadas de Matematicas. Instituto de Matematicas, UNAM,

2010.

[6] R. Bulajich Manfrino, C. J. Rubio Barrios. Olimpiadas en SLP, elemental. Cua-

dernos de Olimpiadas de Matematicas. Instituto de Matematicas, UNAM, 2011.

[7] R. Bulajich Manfrino, C. J. Rubio Barrios. Olimpiadas en SLP, avanzado. Cua-

dernos de Olimpiadas de Matematicas. Instituto de Matematicas, UNAM, 2012.

[8] J. A. Gomez Ortega, R. Valdez Delgado, R. Vazquez Padilla. Principio de las

casillas. Cuadernos de Olimpiadas de Matematicas. Instituto de Matematicas de

la UNAM, 2011.

[9] A. Illanes Mejıa. Principios de olimpiada. Cuadernos de Olimpiadas de Ma-

tematicas. Instituto de Matematicas de la UNAM, 2011.

Page 83: TZALOA Revista de la Olimpiada Mexicana de Matematicas ... · de un tiempo, Sissa, le presento´ a su rey el juego que le hab´ıa pedido: el ajedrez. Lue-go de entender el juego

Bibliografıa 75

[10] Loren C. Larson. Problem-Solving Through Problems. Springer-Verlag, 1983.

[11] I. Niven, H. Zuckerman, H. Montgomery. An Introduction to the Theory of Num-

bers. Wiley, 1991.

[12] M. L. Perez Seguı. Combinatoria. Cuadernos de Olimpiadas de Matematicas.

Instituto de Matematicas de la UNAM, 2010.

[13] M. L. Perez Seguı. Combinatoria avanzada. Cuadernos de Olimpiadas de Ma-

tematicas. Instituto de Matematicas de la UNAM, 2010.

[14] M. L. Perez Seguı. Con la colaboracion de L. M. Garcıa Velazquez y M. Rag-

gi Perez. Matematicas preolımpicas. Cuadernos de Olimpiadas de Matematicas.

Instituto de Matematicas de la UNAM, 2011.

[15] M. L. Perez Seguı. Teorıa de numeros. Cuadernos de Olimpiadas de Matematicas.

Instituto de Matematicas de la UNAM, 2011.

[16] P. Soberon Bravo. Combinatoria para olimpiadas internacionales. Cuadernos de

Olimpiadas de Matematicas. Instituto de Matematicas de la UNAM, 2010.

[17] L. Shively. Introduccion a la Geometrıa Moderna. Companıa editorial continen-

tal. Mexico, 1972.